Sie sind auf Seite 1von 79

2/4/2011Brisbane

O&G
Station 20

Station 5 GP setting: Young female around 30 years old complains about urinary problem and she has history of two vaginal deliveries. Tasks: Take history Ask for physical examination findings Manage your patient

This is a modified AMC book case 128. The differences were she only had features of stress incontinence, denied smoking, had history of constipation. She was on POP but otherwise healthy. I did P/E per book case and there was no prolapse. Then I recommended life style management first including pelvic floor exercise, wt reduction and constipation management. The role player asked about medication and surgery. I told her that some medication such as oxybutynin could help for those who had urge incontinence but not for her. Surgery was not recommended for her at this stage.

AMC feedback: Urinary incontinence (passed)

Station 11 GP setting: middle aged female patient has been trying to conceive for last 12 months but no success. Your task is to counsel the patient. This was an old recall of female infertility. The patient had infrequent sex 12 per month and no history of STDs. Although infertility was more likely due to infrequent sex, you still need to know about all the investigations. The role player asked about investigations for the examiner.

AMC feedback: Primary infertility: unknown cause (passed)

PAEDIATRICS
Station 2 GP setting: father brought his 5 yr old boy due to excessive thirst and frequent urination. BGL is very high (30). Tasks: Explain to the father your diagnosis Short and long-term management This was a combination of f DKA and book case 20 (Refer to old recalls). The role player asked lots of questions which require good understanding of DKA (Study Dr. Wenzels note well).

AMC feedback: Newly diagnosed child diabetes mellitus (passed)

Station 10

Station 16 GP setting: 5 weeks old baby is brought in by his mother due to refusing feeding and SOB. Tasks: Take history Ask physical examination finding Manage the patient

HOCP: Mother said her baby was not feeding well for last week and got worse today. He refused feeding and had difficulty breathing today. Denied cough, fever, rash and diarrhoea, but had no urine output today.

System R/V: Hes never been sick prior to this. BIND questions: Pregnancy was uneventful and mother had home delivery which was nature vaginal delivery by a midwife. All of the antenatal checkups were normal. No concern has been raised regarding development. P/E (You need to ask specific signs, otherwise the examiner will ask you what are you looking for.) GA: sick looking baby, well perfused, nil signs of dehydration, no cyanosis. VS: RR is 70/min, PR is regular. Sorry I cant remember the rest or maybe they were not significant. ENT: NAD CVS: heart murmur can be heard but cant tell whether it is systolic or diastolic. RS: Bilateral basal crackles Abdo: NAD Lower limbs: no oedema. Then the mother asked me what I think of her baby. I told her that I highly suspected her baby had pulmonary oedema due to possible heart failure. I mentioned urgent transfer to ED and further investigation Echo to confirm the diagnosis. The possibility of VSD had been discussed with her and her baby might need surgical repairing for that. Reassurance had been given to the mum that the result of surgery was good.

AMC feedback: respiratory distress (passed)

Psychiatry
Station 7

Station 19

Surgery

1.GP setting: A patient came back to for US result : aortic artery is 5.3 cm in diameter and he is planning for a trip for 4 weeks Tasks: Explain the result Manage the patient This is a straightforward case of AAA. Please refer to previous recalls.

AMC feedback: Abdominal aortic aneurysm (passed)

2.You are a resident on a surgical ward. Your patient is 55 years old female who is day 5 post-hip replacement. She had heparin for 2 days post-op and fluid intake is 3 L in last 24 hours. Now complains of SOB. Tasks: Take history Ask for physical examination findings from the examiner Order investigations Diagnosis and DDx

Hx: She complained of SOB for one hour and there was also chest tightness, which were getting worse. Denied cough, cuff pain, sweating, palpitation, dizziness. She mobilised after the operation. System R/V: not significant PMHx: arthritis Not on medication or allergic to anything. S/A/D: Nil P/E: GA: patient is in distress. VS: BP: 100/60, T 37.5. Pulse regular and the rest are unremarkable

CVS/RS/ABDO/Lower limb: NAD Wound/ IV site: Nil sign of infection Urinalysis: NAD Investigations: FBE, U&E, ESR, CRP, blood culture, cardiac enzymes, Ddimmer ECG, CXY, CTPA or VQ scan Once I mentioned ECG, the examiner handed to me the ECG which was not significant. I told the examiner that I highly suspected PE and I didnt have time to mention my ddx, which I think the examiner got it from my history taking and investigations.

AMC feedback: Chest pain and dyspnoea (acute) (passed)

3. GP setting: A male patient c/o leg pain after walking about 200 meters and he is on ACEI for his HT. He is a heavy smoker as well. Tasks: Perform the lower limb examination on the patient Manage the patient This was a repeated case of PVD and the role player was a real patient, who appeared in our recall from Brisbane for last May. I did the examination per Talley OConnor. I couldnt feel popliteal pulses and Burger test was positive. I mentioned neurological examination but the examiner told me there was no need to perform it. I wasnt entirely sure about the examination findings and I didnt have time to tell the patient to start low dose Aspirin.

AMC feedback: Leg cramps on exercise (passed)

Medicine

1.GP setting: 35 yr old female patient comes to see you for blood result. Her father has cardiac problems and mother has DM. A list of blood result was given: Chol 7.5 HDL 1.2, BGL Normal and the rest cant be remembered. Tasks: Explain the result Manage the patient This was a repeated case. After explaining the result, I advised her regarding life style management and f/u in 6 weeks. If the second blood test in 6-week time was still high, medication would need to be added.

AMC feedback: Hyperlipidemia (passed)

2.GP setting: 55 years old female patient c/o SOB ( this is the shortest stem in my exam) Tasks: Take history Ask for examination finding Manage the patient

This is a repeated case of pleural effusion DDx case. Please refer previous recalls.

AMC feedback: Shortness of breath (passed)

3. ED setting: Male patient c/o indigestion which turned out to be chest pain. Refer this case to book case 65.

AMC feedback: Acute myocardial infarction (passed)

4. GP setting: middle aged female patient was diagnosed Essential HT by your colleague and treated with ACEI. She stopped the medication herself and today her BP is 160/100. Tasks:

Explain the complication of HT Manage the patient This was a new and strange case. At the beginning, I tried to clarify to the examiner whether the secondary causes of HT were excluded or not, but the examiner said all the information were given and she couldnt provide anything more. I also asked the patient whether she had tried life style management before starting medication. She said no. I really had doubt at that time, but I chose to trust my colleagues previous diagnosis. I asked the patient why she stopped the medication. She said she stopped for 6 months due to running out of prescriptions. At this point, I got the conclusion that the patient didnt take HT seriously. Then I talked about all the complications of HT and told her to continue the medication. Also her BP had to be monitored frequently and she told me that she could monitor it at her work place. She seemed satisfied with my answer, but I failed this case. This case might appear simple but I think there could be traps. Please refer other peoples recalls.

AMC feedback: Non-compliance with anti-hypertensives (failed)

5. GP setting: 17 years old male comes to see you due to skin rashes. There are two pictures hanging next to the stem which are from MCQ book. Tasks: Take history and management This was a case I studied a long time ago. I know the diagnosis was acne vulgaris when I saw the pictures, but I couldnt remember the name of medication for it. After taking history, I talked about skin care and offered medication for 3-6 months. The role player asked me whether the medication was effective and I gave him a positive answer. The problem was that I still had some time left and the examiner asked me the name of the medication. My mind was blank and I told him that I couldnt remember the name. I found a MIMS on the table next to me and asked whether I could use it. The examiner was nice and allowed me to use it. But I couldnt find the name of the medication before the bell rang.

AMC feedback: Acne vulgaris (failed)

I could have done so much better when I think back but I am very happy about the result because in the exam my performance was not 100%. I would like to thank everyone who helped and encouraged me during my exam preparation. It is nearly impossible for me to pass the exam without the support from my family and friends. I would like to thank my VMPF bridging long course tutors and particularly Dr Wenzel, whom I have known for nearly four years. He is such an unbelievable doctor who has done so much for IMGs and set an excellent example for all of us.

26th February 2011 Adelaide

1. 56 y old gentlemen come to your GP for a consultation and advice. Take focused history, explain you Dg to the patient and organise further investigations. When Ive started taking history the patients first complaint was that he has some unspecific upper abdominal pain and he thinks is because he has gallstones. Apparently he had a friend who has his gall bladder removed and this prompted him to go and have his gall bladder checked and was told that he has gall stones. On further questioning he never had abdo pain or discomfort, or bloating, or jaundice or any symptoms that might have suggested gall stones. About the pain- unspecific, upper abdo, relieved by glass of milk. PHx: nothing specific, usually fit and healthy FHx: nothing significant Meds: no regular meds but has been taking Nurofen lately because he hurt his back when trying to lift something and was given Nurofen by some other Dr. My Dg was peptic ulcer or gastritis and I based any further Ix around this Dg, so Ive mentioned that hell need r/v by gastroenterologist, he will also need gastroscopy, will have to stop taking Nurofen. As he was persisting on the gallstones issue I tried to draw a picture and explain where they are and that number of people have them and are completely asymptomatic and most of the time are not cause of any problems. The bell rang as I was half way through. Some other candidates mentioned PPI, guess its a good thing to do, I didnt because Mx was not in the task. AMC feedback: Gallstones or Peptic ulcer PASS

2. Middle aged male pt, comes for foot examination and further advice/management. This was real pt and did not have dorsalis pedis pulses on one leg PHx: T2DM

It is the diabetic foot examination station, everything will be provided, the examiner will rush you through the examination just so you could have more time to counsel the patient and suggest further investigations and management. The main point is to ask when was he last seen by endo/physician and request HbA1c, FBE, UE, LFT, ophthalmology and nephrology r/v. I personally think that my physical examination was very poor but guess I did well on the management part. AMC feedback: Type 2 diabetes mellitus PASS

3. Young patient who is known to be HIV positive comes to your GP with a complaint of severe cold like symptoms and cough for the past few days. The role player was coughing the whole time, but was really good, he told me that he has been like that for a few days, and has stoped working (works as a chef in a restaurant). On questioning he will tell you that he is HIV positive for the past 5y but hasnt had his CD4 checked for several years as has been feeling well. Think this was a counselling station where we were expected to tell him that he probably has Pneumocistis carini chest infection (if you dont mention the pathogen he will specifically ask for it). I also mentioned that he needs to have blood test done to check CD4 count and assess progress, he also needs a referral for an infectologist for proper assessment of AIDS and start of antivirals. He will also ask about work arrangement, should he tell his employer AMC feedback: Fever in immuno-compromised patient PASS

4. 12 year old girl run the school marathon and fainted once she finished. Now in ED, you are actually talking to the mother who is worried and wants someone to explain what is the cause and what is the further management. Take history, ask for findings and answer questions. When you start asking questions the mother will tell you that she is perfectly fit and healthy, no issues, normal delivery and development, nothing specific in home, has other siblings, all fit and healthy. She did prepare for the marathon but usually eats healthy and whatever mother prepares, not obsessing about diet When asked about the collapse she said once the girl finished she set down on the ground to rest and collapsed when tried to get up. Mother will also mention that she was very anxious about the run, so didnt eat her breakfast or drink much before the start. When you ask the examiner for findings hell say BP 110/80 and 90/60, BSL 3.5, HR 78 afebrile, no heart issues, no focal neurology. My Dg was just a simple reaction to stress, overtiredness resulting in a postural drop. I suggested short stay in ED, IV fluids, give her something to eat and drink as BSL was borderline, have ECG, check bloods and will r/v in 1-2 hours. Some of the other candidates mentioned HOCM, not sure if this is right or wrong, I didnt as it looked to me as a simple vaso-vagal reaction, having said that I never asked about family history of heart problems, sudden death ect. No harm in doing that AMC feedback: Adolescent vasovagal syncope PASS

5. Picture on the wall of venous ulcer. The pt is a middle aged female who comes to your GP as she has this annoying ulcer on her leg for few weeks, its not healing and she thinks its getting even worse. Take focused history, ask for findings and suggest further management. On history taking she has T2DM and had DVT on the same leg wile ago. She hurt her self while gardening. When asked for findings told as you can see on the picture, than I asked if there were any pulses yes, any discharge yes. My advice antibiotics, take swab and regular dressings. Explained that its due to the diabetes as well as the previous DVT, will take long to heal and the best management is to prevent infection, have good circulation and do regular dressings. Will do one now and will organise district nurse to visit regularly at home. AMC feedback: Chronic leg ulcer PASS

6. Young female, 10 weeks pregnant comes to your GP practice complaining of abdominal pain and vomiting. Take focused history, ask examiner for findings, answer questions and organise further management. On history taking shes being felling well up until yesterday, pregnancy was spontaneous, expected, no issues until now or nausea. The pain is in the lower abdomen unspecific, noncramping, dull, she also has pain in her left loin and had burning and stinging sensation when passing urine. On findings febrile, HR 80, on MCU proteins, blood and leucocytes, office pregnancy test positive. She looks flushed and feels sick. On palpation abdomen is soft, bowel sounds present, tenderness in lower abdo and loins. My mistake I forgot to ask for vaginal examination (exclude ectopic). The examiner tried to prompt me and asked is that all you need to know????? My response looks like upper UTI, likely pyelonephritis, explained what it is, that she needs to be admitted for urine cultures and IV antibiotics for a 1-2 days and than changed to orals. I asked if she is allergic to anything and gave her the possible treatment options. Also mentioned that in ED they might decide to do an US. Again the pt tried to prompt me by asking what will they look for on the US? My stupid response- changes in the kidneys, signs of hydronephrosis.. Questions form pt will the antibiotics hurt my baby, what antibiotics will I need, for how long??? AMC feedback: abdominal pain in early pregnancy PASS

Still surprised how I passed because in my view not excluding ectopic is a critical error, but maybe because I did refer to hospital and mentioned US???

7. You are the HMO on the ward, a med student has just done an MMSE 17/30 and wants to talk about the results. The pt is a 40 something female who scored poorly on concentration and recall. There will be hips of questions, but the main points are:

Cant establish Dg solely on MMSE, only a screening test for cognitive impairment; will probably need to repeat it; dementia frustrated if they cant concentrate and answer properly; depression dont care; delirium pore score on orientation; if suspecting frontal lobe/vascular dementia add more tests (motor repetition, name as many objects/animal in 1min, similarities and differences, all in AMC book). AMC feedback: Teaching a Folstein MMSE PASS\

8. Father of a 1.5y old child comes to you because his son is coughing for more than 10 days. Ask history and answer questions. Cant remember more from the stem but is a typical Pertussis station. Questions what is it?, How did he get it? How will you diagnose/know for sure? How do we treat it? What about us and our 4y old? Can we go and visit friends? For how long do you treat? Make sure you ask what the cough looks like, changes day or night, is the child playful and cheery when not coughing, who looks after child, is he in child care, is he eating and drinking.? Also make sure you advice about feeds, stop when coughing, risk of aspiration, make sure is not vomiting, treatment with Clarithromycin for 7 days, I actually said 7-14, same for family, other child vaccinated and will probably have only mild symptoms, better if not visiting for a while because he is contagious. AMC feedback: Pertussis PASS

9. Young female, think it was 25, comes to your GP because she found a breast lump in her left breast. Take focused history, ask for findings, advice further management and answer pts questions. Typical breast lump question and the examiner was really nice, he actually greeted me infront of the door and said this is an easy one. On history she only found it, not painful its on the external upper quadrant, she is quite anxious as her mother had breast cancer, shes otherwise fit and healthy not on medications. On examination there is a 1cm lump that is mobile, non tender, not fluctuating, not red or inflamed.. Basically have to advice FNAB and US because she is only 25, advice on risks because of family history and explain further management (yearly checks, self examination every 2-3 months and mammography after she is 40). AMC feedback: Breast lump (fibroadenoma) PASS

10. 32 weeks pregnant female comes for an antenatal check-up, she just found out the baby is in breech presentation. Talk to the pt, explain and answer questions. Straight forward station, she said everything was OK so far (must ask about all antenatal screening/visits/test), she has another child (vaginal delivery). She doesnt know much about breech so have to explain what it is, the three types of presentation and the options available is (trial of vaginal if legs fully extended or fully flexed, or caesarean if footling).

There is always a risk of slow progress with vaginal trial so must be psychologically prepared that may need CS eventually. There is also the possibility of external rotation latter in pregnancy. I started by asking her does she have any preferences about the delivery and think the examiner quite liked that. (There was an observer in the room and they kinda nodded to each other and smiled ) AMC feedback: Breech presentation PASS

11. 35 y old male comes to you GP with a macroscopic haematuria. He also had a sore throat few days ago. Take focused history, ask examiner for findings, advice further management and answer questions. This was a really confusing station, first because it looked like PSGM but it was an older pt, and according to the numbers it was supposed to be a peads station. On history taking he is usually fit and healthy, this happened all of a sudden, no pain just blood in his urine, no fever, but he feels that his face is a bit puffed. He had sore throat for couple of days, still soar but didnt see anyone about it. My differentials were PSGM, maybe trauma, stone, some sort of nephropathy. I was leaning towards PSGM so focused all my further instructions in that directions. Told him that although macroscopic hameaturia is unusual for PSGM he still needs admission for full investigation, check MCS, culture, throat swap, blood FBE, U&E, C4, C3, ASOT, US, strict fluid balance, BP control, maybe antibiotics depending on swap and blood results. He wanted to know what is it and I did my best to explain PSGM, the examiner asked me to summarise what investigations I want once admitted. AMC feedback: Haematuria PASS

Pls note that one of the other candidates said that the pt told him this was his second presentation and he felt it was IgA nephropathy. Maybe you should also have a look at other recalls.

12. Psych station, young man comes to your GP because he wants a referral for plastis because he thinks that his nose is too big and is the reason why he cant progress in life. Funny station because he actually took a small mirror from his pocket and was trying to explain what is wrong with his nose. The funniest thing about this station was that the role player actually did have big nose Anyway, this is question from the AMC book, have a read there, but the main think is that you should avoid giving him a referral and suggest psych r/v, family meeting.. AMC feedback: Body dimorphic and social anxiety disorder PASS

13. Middle aged female comes to your GP because she has problems with the nerves. Take focused history, ask examiner for findings and advice further management. Typical thyrotoxicosis/hyperthyroidism station. Not much I can tell on history try to exclude psych issues, menopause, always ask for bowel motion, sweating, weather

intolerance, appetite, weight gain/loss, palpitations, ask for family history of autoimmune conditions Mx check ECG, bloods, FBE, UE, TFT explain that will most likely need meds for a prolonged period of time. AMC feedback: Nervousness PASS

14. Mother of a 9 months old baby comes to you for a regular check up, she wants to know if her son is progressing as he should. Take history, draw growth chart and discuss any issues/answer questions. On history the child is fine, eating and drinking, if you ask milestones they are OK for the age, she says that he is a bit agitated sometimes, but generally OK. When you actually do the chart will realise that up till 3 month right in the middle for weight, but latter there is a progressive decline, and now on 9 months he is on the lowest border, only 10%o. When asked about his food and any diarrhoea, he eats everything including bread, toast, cereals, ect; started solids from age of 4 months, he does have few loose bowel motions every day but she copes well with that and thinks hes not distressed by that. I thought it was Celiac disease and focused my further Mx in that direction. Tried to explain what CD is and why I suspect this is the case and how we make the Dg. Told her that he needs an assessment by peads, he will need specific antibody tests and if positive will need special diet. I also explained that it meant he has a life long condition that can be easily managed by diet, but if not managed can lead to serious complications. At the end I asked if she has any questions and she said its too much to digest. Now looking backwards think I should have given her more time to absorb all info and ask if she has questions, or if anything is unclear, instead I just kept talking for few minutes. AMC feedback: Infant with poor weight gain PASS

15. Middle aged pt is admitted to ED with confusion for more than 12h, now he is incomprehensible and you are supposed to take history from his wife, ask for findings, give differentials and explain questions, dont talk about Ix or Mx. Really weird station, not much I can tell because I failed this one. On history nothing much, all of a sudden he become confused, non drinker, usually healthy, no trauma. Findings HR90, T 38, everything else is normal (no bloods, no CXR, no CTB.) My differentials were trauma, delirium, infection; than she asked what kind of an infection, I did mention acute brain sy, meningitis, chest, maybe even a UTI presenting as delirium. However I didnt mention encephalitis AMC feedback: Encephalitis FAIL

Please note the candidates who did mention encephalitis were asked how he might have gotten it?

16. 18 y old girl comes to your GP because she wants Implanon. Take history, advice and answer question. Again not much I can say as I failed this station as well. These were the most dreadful 8 minutes in my life as I knew I will fail even before I entered; I knew nothing about Implanon. The only thing I knew was that its a rod inserted on the inside of the nondominant arm and lasts for 3y. I was so confused and uncomfortable, so when the role

player asked me if she can have it done today I forgot even what I knew that its a surg procedure and shell need to be booked in. AMC feedback: Implanon counselling FAIL

My advice read everything there is about OCP and HRT because you WILL get a question about one of the two

So there it is, I passed 14/16, quite happy with the outcome. I only studied actively for 1 month, did not get much time off from work so had to use my annual leave. Ive been through the AMC book and the recalls 2008-2010. I did try to study around the topics a bit, not just memorise recalls. Thats why I was a bit stingy with this recall; think its always better to prompt candidates to actually look for more. I only used Mourtagh and Oxford and think its more than enough. Few tips: dont just read recalls, broaden your knowledge by reading a bit more because topics are the same but questions wary slightly be confident. I actually made some mistakes and missed some important things on examination, but maybe because I was confident in my differentials, Dg and further Ix and Mx think I still passed some stations if you cant find a study group dont stress, I had to study alone ( have a 2y old boy and had to run back from work, than there was always something on the weekend, so couldnt really fit with a study group). However always try to speak out loud, when you just go through the questions in your mind youre not practising and will most definitely be short in time, or will forget something. Wenzels classes are also wonderful, unfortunately a bit difficult to attend to if you are working. Nevertheless if you have a chance go to at least few, Ive only been to 3 but found them to be really useful

All the best

CLINICAL EXAMINATION MELBOURNE 19TH MARCH 2011.

1. You are in GP clinic. A 52 yo man who has smoked 20 ( or 10) cig/day for 20 years, come for check up because of chesty cough. - Perform respiratory examination. - Summarise the case to the examiner.

This is my 2nd station ( my 1st station is Rest station!). Outside the room, when I tried to outline what I will focus when doing physical examination I only reminded

myself: don't forget signs of Lung cancer ( Horner's syndrome, hand examination for nerve root compression...). And I totally forgot COPD.

I entered the room. After greeting the examiner and the Pt, I made a quick glance to the Pt and try to guess if this was a real Pt (but I could not). There are no abnormal signs on PE but nicotine stain on right fingers. By mistake, I checked vocal resonance with a stethoscope! Fortunately I realised it was wrong so I checked it again with hands. At the end, when I was mentioning Horner syndrome' signs, Pancoast syndrome's signs the examiner rushed me to check PFM. In a hurry, I connected the mouth piece to the device, asked the Pt to do PFM. He did it wrongly. I asked him to repeat. Time almost ran out. I ran the commentary quickly and ordered some Ix: chest X ray, CT scan.... The examiner suddenly asked me to wash my hands. I was so panic and wondered if i missed anything. Anyway I still washed my hands and the bell rang.

AMC feed back: respiratory examination.

2. A 24 yo student comes to your clinic, c/o headache which started this morning. - Take relevant, focus Hx. - Find out PE from examiner who only gives you the clinical findings which you're asking for. - Give provisional diagnosis. - Mx.

Outside the room, I thought maybe it is tension headache, migraine... But when I came into the room. The Pt is a young man lying on the bed, suddenly I changed my mind: is it SAH?

I greeted the Pt then asked if he needs pain relief. The examiner said: no, he's OK. "PAIN" questions: Pain 3-4/10. No specific type or location. No neck pain. No other symptoms. Past Hx of migraine, 3-4 times last year. This pain is different from the previous ones. No Hx of contact, no stress related, no head injury.

PE: No rash. T 37.5, BP normal. FO: not available. Neck stiffness +. When I asked for Koernig, Bruzinski, Babinski signs, the examiner looked happy.

Provisional Dx: Meningitis maybe due to Meningococcus, HI. I told the Pt he needs to admit to hospital for further Ix and management. He was not allergic to PNG so I gave him PNG. I mentioned about notifiable disease and prophylactic ABT for close contact. The examiner smiled and asked me who is close contact. I turned to the Pt: whom do you live with? Girlfriend! Then I said his girlfriend needs Rifampicin for prophylaxis. I asked if he had any concerns he said no. Then examiner asked me to go out.

AMC feed back: Acute headache.

3. 4 yo girl was discharged from hospital 3 days ago after an asthma attack. She is fine now but mother wants to know about asthma action plan. She comes to ask about it at your clinic. - Discuss about the girl's condition. Explain Asthma action plan. - Future Mx.

When I entered the room I saw asthma action plan on the table. I was very happy and I grabbed it. To be honest, at that time I was so confused and could not trust my memory ( !) although before the exam I tried to memorise asthma action plan.

I asked the mother about her daughter's condition, how she is now, how she goes with the medication... The mother asked me: which medication? I said the tablets (I means the corticosteroids orally). She replied: all is OK. I said if I can ask her some Qs. Then I asked Hx of asthma of her daughter: when Dx, how frequent, medication.... Trigger factors, Fa Hx... The mother was also asthmatic but has settled now. I started to expl what asthma is ( draw the pic). With the asthma action plan in hand, I said this is symptom based plan the stared from WELL, UNWELL....When I finished I asked if she can follow me she replied so far it's OK. I explained about reliever, preventor ..... Then I said at this stage your daughter only needs reliever medication (because she has only 2 attacks in the last 6 months). I asked if she gave the medication via spacer or puffer and if she know

how to use it. She said: spacer but not sure how to use. Then I said via space is the best way to give reliever at this age and showed her how to use it correctly. She ask how about her daughter's condition in the future? I said she may grow out of asthma as your case. Then she smiled. Finally I reminded the mother to avoid trigger factors and to realise red flag symptoms. Bell rang when I just finished.

AMC feed back: Chronic asthma.

4. A 64 yo lady coming to your clinic c/o lower back pain. She was Dx with chronic lower back pain for years. Recently she has groin pain. - Take Hx. - Give Dx.

When I read the senario, i was so nervous whether it is 2 symptoms of 1 disease or 2 different diseases. Suddenly bell rang and I had to enter the room.

The Pt was sitting on the chair. I asked if she need pain relief. She said no. PAIN Qs: lower back pain for years, did not getting worse recently. Right groin pain for last 4 days. Appears when she walks for a distance. Settle in few minutes after she stop walking. No calf pain. No lump in groin. No other symptoms. No other medical condition. PE: Everything was normal. No tenderness on tenderness along the spine. No muscle wasting, no swelling, no deformities on right leg, Knee normal. Trenderlenburg +ve (so it could be : OA, gluteus medius weakness, NOF fracture). Thomas test -ve. Examiner asked what Thomas test means check fixed flexion deformity of the hip for ex in OA.. I was panic because I could not reach to the Dx. If it is OA? but why OA has the symptoms like intermittent claudication. If it is sciatica? but the Pt confirmed: no pain at the back of thigh or leg, no abnormal sensations. Then I asked for Burger test, the examiner said it's not available. When I was ordering for Ix: X ray, Doppler US.. the bell rang!

At the rest station I kept thinking about the case. My performance was not good, I took a lot of time for Hx as I could not find any cue for Dx. I felt so upset about myself. Anyway I reassured myself I have a right to fail 1 in 4 stations! :-)

AMC feed back: hip pain.

5. The long Hx of the elderly lady who has been Dx with schizophrenia. Recently she refuses eating, taking medication... Medical Tx has failed. The psychiatrist decided to have ECT for her condition. Her son comes to your clinical asking for ECT. He has been given a consent from his mother. -Expl ECT. -Discuss with the son and answer his Qs.

I said usually we do not dislodge one's medical condition to others. At the time, the examiner looked at me nervously as if I did not understand the task. I understood his feeling then I continued: as your mum has given you a consent so I will talk to you about her condition and Mx plan. I could see the relief on the examiner's face! Then I did the case as Dr Wenzel's notes. Firstly, I said about indication of ECT. Then I told him the whole procedure: anaesthetic assessment, Pt will be under GA. Some electrodes will be put on the scalp and electrical current will be used for Tx. During that time, Pt may have jerking movements, don't worry. She wont feel them at all because she is under GA. He asked me are you sure she is not in pain. I said yes. Then I said about SE: headache, dizziness, forgetfulness. But they are temporary. You see, now your mother is not in a good condition, she becomes self neglected. She refuses to eat and drink, her health will be deteriorated soon. After assessing carefully we think ECT is the best choice of Tx now. .He looked happy. I added if we compare the benefits outweigh the disadvantages. I finished early. The examiner asked me: if I finish my task then I can go out. I was not sure whether my performance was bad or I missed anything. Then I replied my task is discuss with the son so I would like to stay here if he wants to ask me more. They both laughed. Then the examiner asked me you'd better to go out.

AMC feed back: ECT.- procedure, indications, effects.

6. A 70 yo lady c/o back pain. X ray was done and showed crushed fracture at L4. Bone density -3. She comes today for the result.

- Discuss, expl the results. - Plan Mx.

I greeted the Pt then asked if she is in pain. She said she OK. Then I asked Hx - Any trauma, injury. - Menopause: when, any symptoms of menopause. Still sexually active, any complaints. Pap smear, mammography... - Malignancy symptoms; night sweat, weight loss, lumps in the body... - Diet: milk, diary product. - Exercise, sun expose... - Smoking. - Fa Hx. I explained her that there are 2 issues: - Spinal fracture: pain Mx, rest on firm mattress... physiotherapy, occupational therapy. - Bone thinning: Needs more Ix to r/o other diseases: RF, multiple myeloma, hyperparathyroidism. Then I mentioned about life style modification: stop smoking, healthy diet, take diary product, exercise... Give some medication: Vit D, biphosphonate ( how to use, SE..), SERM. I did not mention about HRT because her menopause was 20 years ago!.

AMC feed back: osteoporosis.

7. A elderly man had operation few days ago for bowel obstruction. The surgeons found that there is a tumour in pelvis and some spreading to mesentery, liver. They decided to leave the tumor and did colonostomy. The biopsy post op has confirmed colon carcinoma. The Pt now comes to your clinic c/o abdominal pain and ask about his condition. Answer the Pt's Qs.

Outside the room, I wondered if this is breaking bad news or if it is complication post op because the Pt c/o abdominal pain. I asked the Pt how severe the pain is, if he needs pain relief, any other symptoms.. he said no. At that point I though it would be a case of breaking bad news. Then I asked if he comes alone... When I stopped for a while for his relief, surprisingly he pushed me to continue. I said your case will be Mx by MDT, the oncologist will arrange further Ix: CT scan, bone scan to find out where the cancer spread to. About the pain, there are a lot of options depending on the severity ( I mentioned about step wise of pain Mx in J.Murtagh book). He asked me how long he can live. I said it's too early to say, be optimistic, the quality of life is more important the quantity of life... He insisted he wanted to know how long he can live. I was reluctant to say but at that time the examiner told me loudly: how long he can live? I was so panic. I turned to the examiner and said for colon cancer at this stage, I mean Duke D, 5 year survival rate is 25 - 30%. Then to the Pt, I said we will try our best to give you a comfortable life and the outcome would be different from person to person. Finally the bell came. I escaped the room.

AMC feed back: Carcinoma of colon ( recurrent, advanced)

8. At the small countryside hospital, 12 month old child suddenly refuses to stand on his legs when his mother holds him by his sides to support him to stand as usual. The child was healthy previously. Father brings his child to the hospital. - Take Hx. -Ask PE - DDx. - Mx.

All Hx about pregnancy, delivery were normal. No flu like symptoms recently. No trauma, injury. Immunisation up to date. Father had no idea about where the pain is, any redness, swelling... PE: T 39. Crying when touched to right leg. Swelling at right knee. Not sure about effusion as child did not cooperate. My provisional Dx is septic arthritis. However I needs to rule out osteomyelitis. Refer the child to the tertiary hospital where the paediatrician will do further Ix; FBE, blood culture, knee aspiration for microscope, culture, bone scan. We also start with IV ABT.

Examiner asked why do you send the Pt to the hospital? you are in the hospital now. I did not forget the senaerio, I replied this is a countryside hospital, I'm afraid if we don't have the facilities to follow up the case as many severe complications may happen esp septicaemia and the child is only 12 month old. How long for ABT? Because the child < 5 yo, ABT should be 3 weeks IV when then 1 week orally ( I'm not sure) When I asked the role player if he has other concerns, the examiner asked me: do you think you have finished your task. He did not look happy. I took a quick glance to the paper task and found that I did mention all the tasks! Why did he look so cold? As I had no more information to talk so I turned to the role player again and asked if he could arrange for his child's admission. He said yes. I said I will arrange NETS ( National Emergency Transport Service) to transfer the child to the tertiary hospital and also social worker will support him if anything in need at home. Examiner then asked me: where does the bacteria come from? Well, they could come from skin infection, throat infection, sometimes we are not able to find out their origin. So next time if your child has skin lesion, sore throat... pls bring him to see the doctor.

AMC feed back: Septic arthritis.

9. A 50 yo man, chest pain. Last time he had stress test done which showed ST elevation ( I forgot which leads). Today he comes for the results. His records are: BMI 35, BP 130/80, Cholesterol 6., smoking 10 cig/day. - Expl to the Pt his test results. - Plan Mx.

I asked if he is OK, any chest pain now. He said he is OK now. PAIN Qs. Risk factors of CVD Qs: diet, body weight, BP, Hx of DM, hypercholesterol, Stress.. Fa Hx of DM, HT, sudden death... Then i expl about the test and its purpose. I drew the pic to expl coronary artery's function. I said about Mx plan based on life style modification: A( alcohol), B ( BMI, BP), C( smoking), D ( diet), E ( exercise). About medication: statin would be consider if there is indication as in his case if cholesterol increases up to 6.5. The role player asked me: are you sure if I follow your advice I will have a healthy life? I nodded my head and said surely, you will. He laughed. I don't know why whenever they laugh I become anxious and panic. So I wondered what else he wanted. Luckily, he said: do I need an operation? I realised I missed something. I

said it's too early to say about op. I will refer you to a cardiologist he will perform Ix to find out where the blockage is by injection dye. Then he will decide whether you need op with stent or graft. They smiled and asked me to go out.

AMC feed back: Angina pectoris.

10. 4 yo girl whose parents have separated for last year. For the last 4 days the mother has noticed rash in her vulva after the child came back for her father's place. The child also refuses to come to her father's place. The mother suspects sexual abuse. Take relevant Hx. Ask examiner PE. Talk to the mother about her concerns.

The role player was a Chinese medical student. She looked really upset. After I greeted her, she started sobbing. Then she continuously told me about her suspicion, blamed her ex, asked me to help her to keep the child with her, prevent the father from doing his custody.

Outside the room, I thought this is child abuse but at this point I wondered if this is a case of mental disease ? Depression because her acts were exaggerated. So I started asking Hx with HEADS Qs. No specific findings. Then I ask about the child: the only child, was healthy, growth and development normal. This is the 1st time she has this condition. Child stays with father 3 days/week. He does not smoke, drink, has no new partner. No financial problem in both sides.

PE from the examiner: Child looks normal, VS normal. No other signs of trauma: bruising, old fracture, deformities. Rash in vulvar area. No bruise, no laceration, no scratched marks, no vaginal discharge.

Then I expl to the mother that I suspect this is non accidental injury. However I will take the pictures, swab for evidence and Ix to r/o condition likes: infection, eczema which are less likely. The child will stay in the hospital which is the safest place for her at the moment. You can come with her at any time. I also report the case to CPU, CSA... they will help her as much as they can. I will contact social workers

they will help you if there is anything in need. The mother still looked upset and started to cry. She said: you're not going to help me to keep the child with me, don't let her come with her father. Again, I was confused! Why did she still upset after my explanation? Then I gently touched her hand and said pls calm down and think positive. We are here to help you. It is my responsibility to look after your child's health. The CPU, CSA will try their best to solve the problem. Event family court may involve. She was still upset but the bell rang and I have to go.

AMC feed back: Sexual abuse.

11. In GP clinic, a 24 yo female student, c/o RIF pain for last few hours. - Take Hx. - Ask examiner PE. - Provisional Dx, Ddx.

After greeting the Pt, I asked the examiner if my Pt is haemodynamically stable. He smiled and said: good, she is stable. PAIN Qs: RIF, 3-4/10, no radiation, no other symptoms. 5P: not has children yet, trying to conceive for last 3 months, LNMP 3 weeks ago, lighter than normal, Pap 6 months ago normal. Has no idea about breast US. No past Hx of STD, doesn't know about blood group, no Hx of miscarriage, no Fa Hx of female cancer. PE: T 37.2, BP 110/60. Tenderness on RIF, no rebound tenderness. Mc Burney: not clear. I forgot asking about Rosving's signs, psoas, obtulator signs. Speculum ex: no bleeding, no discharge. Os normal, close. Uterus: retroverted, normal size. Tenderness on RIF and Douglas porch. Urine dipstick -, pregnancy test +. I told to the Pt due to your last period is abnormal, your pregnancy test +, your abdominal pain so most probably you have ectopic pregnancy. I needs to admit you to hospital where the gynaecologist will do Beta HCG quantity and US to find down the location of the foetus/ sac. When the Dx is confirmed there are 2 options of Tx: medical Tx ( I mentioned indication) or surgical Tx. Outcome is very good. I drew the pic and said your chance to have pregnancy is 60%.

The role player asked me if she have appendicitis. I said the DDx are appendicitis, ovarian cyst torsion, PID which are less likely. Don't worry, with further Ix we can r/o the Ddx. I asked if she has other concerns. She said no. I finishes early.

AMC feed back: Probable ectopic pregnancy.

12. 18 or something lady who was admitted to hospital due to DKA. She was Dx DM type I at 15 yo and on Insulin. Recently she usually skipped the meals because she thinks she is fatty.

Discuss about her condition.

Firstly I assured her about confidentially. She looked happy and said pls dont tell my GP! I start with taking short Hx: how is her DM, well controlled? Check up? Any complications. Then I asked about her current condition: is she OK now. And HEADS Qs: no stress, no family conflicts, no any specific thing. She skipped the meals because she want to be thinner. She also self induces vomiting and uses laxative to keep good body image. Then suddenly she asked me: doctor, do you think eating too much and getting fatty are good for health. I smiled with her and said: you are right, we should not eat too much but need to have healthy diet and keep body weight in normal range. Turn to the examiner, I asked if I can ask some PE. How is her BMI? He said she is there, you can see. Then I asks BP, any dropped BP no. Heart rate normal. Any signs of dental decay, callous on fingers..- nothing. No anaemia, no epigastric pain. I forgot to ask about menstruation. I asked the role player if see know about DM I. She said no. I exp what it is ( pancreatic gland does not produce enough Insulin ). That's why we give Insulin inj to control BSL. Due to low BSL after Insulin inj, if you skipped the meals you may suffer from severe complication. Apart from that due to self induced vomiting, using laxative you may have: dental decay, inflammation/ rupture of the foot pipe, callous on fingers and most important is electrolyte disturbance. Loss of potassium can affect to the heart and cause critical condition. Your condition is called bulimia nervosa. It's a kind of abnormal behaviour. I will refer you to see the psychologist who will talk to you ( CBT) and the dietitian who will give your a good advice about

healthy diet. Finally I said I think there are too many information for you today so I print some hand out for you to read at home. Here is my card. You can call me if you have any concerns. Then I asked if she needs to ask anything. The examination told me: you told her everything. On the way I went out, the bell rang

AMC feed back: Bulimia nervosa.

13. A 24 year old lady who is at 10 week of pregnancy came to your clinic to ask about Down syndrome because her cousin just gave birth a Down baby.

- Take relevant history. - Counselling her.

I started by greeting the Pt. She looked anxious. When I was asking history, she seemed to be in a rush to know about Down syndrome. However I still tried to ask full Hx. About current pregnancy so far: This is her 1st pregnancy. No difficulty in conceiving. Has not had antenatal check up yet. Has symptoms of early pregnancy. No abdominal pain, no vaginal bleeding/ discharge. Not yet taking folic acid. No idea a bout blood group. Hx: Medicine: healthy. No DM, HT, SLE, bleeding disorder, clotting disorder. Ob & Gyn: no STD, no miscarriage. Regular period. Pap's normal. Fa Hx: no DM, HT... No Hx of congenital abnormalities except the above newborn baby. Then I asked her if she know anything about Down syndrome. She said no but asked if it is hereditary. I said it is not hereditary disease, it's congenital abnormalities due to abnormal chromosomes. Then I expl about 23 pairs of chromosomes. The 21st have 3 instead of 2 chromosomes. Briefly mentioned about physical abnormalities and consequent complications. The Pt looked really panic now ( I wondered if it is my mistakes to talk about these bad things!)

I quickly reassured her. Nowadays we have a lot of antenatal screening tests as well diagnostic tests for Down syndrome. Screening test: 1st trimester: PAPA, BHCG, US: nuchal thickness at (11 13 week), 2nd trimester: AFF, B HCG, Oestradiol, Inhibin A. Diagnostic tests: CVS ( 9 11 week, result in 2 days, miscarriage 1- 2%), amniocentesis ( 14 16 week, result in 2 weeks, miscarriage 0.5%). She said: if Down baby has confirmed do I need to have abortion? Well Down syndrome is actually not a significant condition. Depends on the type of chromosomal abnormalities: totally or partially ( I wanted to mention about mosaic type.... but don't know how to exp them in lay terms) baby may have a mild or severe symptoms. Good things is they are lovable, compliant . So actually, there is not a indication for abortion in all cases. Finally the choice is yours. She looked relieved. So I concluded: pls take folic acid right now and continue up to 12 week pregnancy. I will arrange all the antenatal tests for you plus some screening tests for Down syndrome. As your Fa Hx, your case will be follow up at high risk clinic. When I ask her if she has any concerns, she said no. I still have time, when the examiner ashed me to go out, suddenly the role player asked: can you tell me the incidence of Down baby in my case? I said: for normal population, before 35 yo, incidence of Down syndrome is 1/700. But I don't know the incidence if the cousin gets Down baby, I will ask my seniors. She smiled the bell rang.

AMC feed back: explanation of fetal abnormality screen.

14. A well known case: a 40 something yo man who had pace maker put in few year ago came to GP clinic c/o tiredness. No alcoholism. LFT has been done : increased GGT, ALP. US of abdomen showed some changes of liver ( I forgot). The GP has written a referral to the specialist for further management. - Exp about the result to the Pt. - Ask the examiner PE. - Which Ix would the specialist order? You will only be given the result of which you ask for. - Give Dx and discuss with the Pt about further Mx. I greeted the Pt then ask how he is today. He said he is tired. Then I asked if he feels comfortable. He said he is OK. Then I grabbed the copy of the senario, showed it and expl to him 1 by 1. I said: GGT, ALP are liver enzymes. The increased levels mean there is some damage to the liver cells. Also US showed some abnormalities in the liver. I said I will ask the examiner about PE to see how the disease is going on then continue talking with him. PE: Pt alert. No confusion ( hepatoencephalopathy), no skin discolouration. No jaundice. VS normal. No parotic enlargement. No spider naevi, no carput medusa. Chest, heart normal. Heart rate regular. Liver enlarged 2 cm below the

costal margin. When I asked about density: hard or soft, about the border: rough or smooth, hepatic jugular reflux... Not clear. No supraclavical LN, no collateral veins. PR: no specific findings. Flapping tremor: -ve. Ix : I asked if I have iron study, the examiner immediately gave it me, ferritin serum increased, TIBC decreased, transferrin saturation: increased. Then I expl to the Pt; most probably he has the condition called haemochromatosis. It is a genetic disease. Due to high level of Fe, there are many complications which depends on where Fe is going to accommodate. For ex: It can affect to heart as your case, affect to joint: joint pain, affect to pancreas: bronze DM, affect to testis; atrophy, affect to liver: liver cancer which may be a concern in your case. Dont worry the gastroenterologist will do biopsy to see if there is nasty growth in your liver. Mx; Giving blood 500ml every week meanwhile follow up iron tests. When the level back to normal, giving blood every 2 3 weeks When I asked if he has any concerns he said no. Then suddenly my memory is switched on, I said because it's genetic disease, it would be a good idea if your sibling, children have a genetic check. The examiner said: you seem very interesting with this topic. Then he ask me if I think I finished my task. I said I'm not sure but my task is to discuss with the Pt so I dont know if he wants to ask me anything. The examiner gave me the other paper with : C282Y, H63D. He asked do you know it. I said they are 2 mutations of gene disorder in haemochomatosis. He looked happy but the role player said: I dont know anything. They both laughed then the examiner said: you'd better go out. I finished early.

AMC feed back: Abnormal liver function test.

15. A 32 yo lady came to your clinic, wanted to know about sterilisation. Husband, 36 yo, is happy with their decision. They have 3 kids ( forgot ages). Counselling. I stared: I knew from the notes you want to ask me about sterilisation. Can I ask you some private Qs before we discuss? She said yes. Then I asked: Period, Pap, Pill, Pregnancy About the pill: what kind, how long, any complication, follow up: BP, weight..., any complaints ( no). Family situation: no family conflicts, healthy children. Marital status: happy. General healthy: good, except operation for complicated appendicitis with a long scar next to the mid line. I asked of she know about sterilisation, from which sources. I drew the pic then explained how sperm and egg meet together. I said the whole procedure of sterilisation. Usually it will be done with keyhole surgery but in her case may be

normal abdominal operation as she had previous op. I emphasised: successful rate is not 100% due to reconnection, and no prevention for STD. At the end of the day if she changes her mind, wants to have baby, needs to have micro operation: successful rate if 60% with hight rate of ectopic. Then I advised if the is no complaint with pill she can continue with it or try others: depoprovera, implants, IUD ( I said about route, advantages, disadvantages of each method) Examiner asked what kind of IUD, I said: Minera, Cooper 375. Finally, I also mentioned about male sterilisation which is more simple. When I ask if she has any concerns, she said no. Then I said you dont need to be in a hurry, go home, think carefully if you want more information I will give you hand out.... I finished early.

AMC feed back: request for Sterilisation.

16. At GP clinic, the young lady suddenly has calf pain. She has 2 relatives diagnosed with DVT. This is the 1st time she has this condition. Take relevant history. Order Ix. Expl to the Pt about Dx. Mx

I asked all risk factors of DVT: no travel by flight, no pregnancy, no pill, no Hx of DVT, or fracture or immobilisation, no varicose, no smoking, no obesity. No night sweat, weight loss, no lumps, bumps on the body recently. Sister had DVT. Cousin also had DVT. Both are fine now. In general, she is healthy, not on any medication esp female hormone. No Hx of other diseases; SLE... Current condition: no SOB, no cough, no chest pain. Ix: Doppler US: thrombosis at level of left lower leg, D dimer, thrombophilia screening test, ANA, coagulation profile....:not available. Epl: I drew the pic of the leg, expl about the role of venous system. Due to clot in vein, blood can not get back to the heart. Trapping of blood causes pain. The most dangerous is this clot can dislodge and travel to other organs: lung ( PE), brain ( stroke)...They are critical conditions. That is why we needs to order further Ix. Thrombophilia screening for Tx: If there is Fa trait, Tx will be long life. Coagulation profile: INR for follow up Tx. Refer to haematologist who will put her on Heparin 1mg BD inj and Warfarin orally. When INR 2-3 stop Heparin and cont with Warfarin. Examiner asked what kind of Heparin, I said low molecular.

Then I mentioned about SE of medication: bleeding gums, red urine, black stools... Needs to come back immediately. And intereaction of medication: pill, Nexium, Antacid.. cause decreased INR, Corticosteroid, Rulide, Aspirin cause increase INR. They looked happy and did not ask me anything. I was about to run to the next station. The examiner said you finished your exam. Good luck! At the time I realised that was my last station!

AMC feed back: DVT

I tried to write in details what happened in the exam so you can roughly know about the real situation. Apart from knowledge, the more appropriate you can manage to response to the examiners' Qs or the role player acts the better outcome you can get, I think. Study hard, practise repeatedly then everyone can make it. Best wishes to all of IMG candidates who is struggling for AMC exams. Please help each other because we still have many other exams. Passing this exam is just the end of the beginning.

AMC Clinical Examination 19 March 2011, Melbourne


O&G
(1) 24 years old girl with her first pregnancy at 10 week gestation came to your GP clinic to ask about Down syndrome. She is concerned about this because her 32 years old sister gave birth to a baby with Down syndrome. She has already decided not to continue this pregnancy if she found that her baby has Down syndrome. Task: take focused history; manage the case and counsel the patient accordingly.
History taking How are you today? The RP said I am good. As far as I know, you want to get information about risk of Down syndrome, is that correct? Yes. Before we get to that point, I would like to ask you some questions, related your pregnancy, and is that O. K? She said o. k.

I asked her how your pregnancy is so far. Have you already seen the doctor and done all blood test? All results were normal. Are you taking folic acid? Yes. Do u have any medical problem that I should aware of? No. Do u smoke or drink alcohol? Are you on any medication? No. Is there a genetic problem in your family apart from Down syndrome, cleft lips or cleft palate? No. Explanation and answer the questions Then I asked her what you know about Down syndrome and explained to her about Down syndrome. It is a genetic disorder that associated with advanced maternal age however it mostly occurs below 35. In general population, the risk of having a Down syndrome baby is 1:600-700 but the risk increase with age and if there is family history of Down syndrome. I explained that there are 2 sets of tests available: one is screening and the other is diagnostic. The advantage of Down syndrome screening tests is that they only require a blood test from the mother and an ultrasound to check the neck thickness of the foetus, so there is no risk to the pregnancy. The first one is triple test (USG, PAPP, and BHCG) which is done during the first trimester (typically at 11 to 13 weeks of pregnancy) and the second, quadruple test is done during second trimester (4 markers in your blood which are either protein or hormone). , if they are higher or lower than normal, it can be suspected for Down syndrome. If both tests are negative, it is less likely to have a Down baby. On the other hand, if both tests show abnormal results, you will be then put in a high risk group and need to have another test done, which is a diagnostic test which is nearly 100% accurate to detect Downs syndrome. There are two tests available which are invasive procedures and require putting a needle into the uterus or placenta and removing some fluid or tissue. Chorionic Villous Sampling which is done at 11-13 weeks and the results will be back in 48 hours. But it carries1% risk of miscarriage , and Amniocentesis which is done at15-18weeks and need to wait for 2 weeks to get the result and which carries 0. 5% risk of miscarriage (1:200). I asked RP do u understand what I am explaining to u so far or do u have any concern? Then RP asked me "Do you still offer me diagnostic tests if the screening tests are negative?" I said, screening tests for Down syndrome cannot tell for certain whether your foetus actually has Down syndrome; rather, they tell you whether there is a low or high risk that the foetus is affected. However, a diagnostic test can tell for certain if the foetus has Down syndrome or not. Since you have family history of Down syndrome and you do not wish to continue the pregnancy if you found your baby carries this gene, I would like to offer you the definitive tests. But the decision to have diagnostic tests depends on your wishes. There is no right or wrong choice; you can decide what is best for you and your family. You can discuss with your husband and I can arrange another meeting with both of you. If you want I can refer you to obstetrician who can explain to you about the chance of having baby with Down syndrome and about the invasive procedures in detail. AMC Feedback: Explanation of foetal abnormality screen (passed)

(2) 22-year-old lady came to ED with right sided abdominal pain since last night. Task: Take history, ask examination finding from the examiner, Order necessary investigations and explain your management plan to the patient.
My DDx of pain in RIF: Appendicitis, Threaten abortion, Ectopic pregnancy, Twisted ovarian cyst, PID, UTI History taking I asked examiner whether my patient is haemodynamically stable or not. asked me again what you want to know. I said BP, PR, T _all normal. I offered the RP painkiller but she said its ok. Then I asked all pain questions: pain started suddenly last night, around right lower part of the abdomen and radiate to the back, 6/10, not relieved with anything, not associated with fever, nausea and vomiting, no discharge or blood from the vagina. I asked about waterworks and bowel motion. She mentioned as normal. When asked about the LMP, it was about 3 weeks ago, regular, moderate amount, and no pain. She is married and not on any contraception because they are trying to conceive. When I asked about the symptoms of pregnancy, she denied. No past history of STD/PID also. Have u ever been pregnant before? She said no. When I asked RP, have u done a pregnancy test, the examiner said it was positive. PE Since I have known the vitals, I went for focus examination of the abdomen and pelvic. Pain in RIF, no rebound tenderness, all tests suggestive of appendicitis negative (no need to ask but want to make sure). No renal angle tenderness. Pelvic examination: no discharge on inspection and os is closed on speculum examination, but on bimanual examination, there was tenderness in the right culs, Ut size is normal. Urine dipstick normal. Quantitative BHCG and USG results were not available. Explanation and Management Then I explained to the patient. According to the history, the examination findings and the test result , there is a high possibility that you have what we call Ectopic pregnancy that is a pregnancy outside your womb, this can be in the tubes between your womb and ovaries as in most cases or inside the tummy, which is very rare. As the pregnancy is not in the usual place, it cannot continue to term. I understand that it is very disappointing for you. He

Now you need to be admitted to the hospital where you will be seen by obstetrician; she or he will make a definite diagnosis by laparoscopy or key hole surgery. First you will be put to sleep, and then they will insert a tube with lens through a small incision in your tummy, and have a look at your womb and tubes. Treatment will depend on the size of the sac, viability of the foetus, level of BHCG. Depending on this, the obstetrician will decide either to inject medication or remove the foetus by operation. Although the doctor will try to preserve the tube, there is still chance that they need to remove the tube in case it is damaged. Do you have any questions that you want to make clear? Can I become pregnant again? Yes, you can as there will be one tube and ovary left even if the affected tube will have to be removed. But still there is a chance of having ectopic pregnancy in your future pregnancy. (If tube need to be removed, there is a chance of having pregnancy every alternate month. AMC Feedback: Probable ectopic pregnancy (passed)

(3) 35- year- old woman came to your GP clinic because she wants to discuss about sterilization. She has 3 children and has been taking oc pills for a long time. Task: take focused history, ask physical examination findings from examiner and counsel the patient sterilization and other contraceptive methods suitable for her.
History taking I introduced myself and said from your notes I understand that you want to discuss about female steralization. Before we come to this point, may I ask you a few questions? RP said its ok. I asked the reason why she doesnt want to continue to take oc pills anymore. She said I do not want to introduce all these hormones into my body and I will continue to be sexually active for another 20 years. So I do not want to use any hormones for more than 20 years. Since I have completed my family, I want to do the sterilization. When was your LMP? It was 2 weeks ago. Can you tell me about your period? It was regular and heavy before taking pills, but now regular and normal flow after taking pills. Have you ever been diagnosed with STD/PID? She said NO. When was your last papsmear? 6months back-normal Do you have any past medical problems? Have you undergone for surgery? Yes, it was a very complicated procedure since appendix was burst. Then she continues to talk about that procedure in detail.

At that time I did not realise that why she kept saying about appendectomy rather than sterilization. Anyway I continue to ask her about her last Pap smear (normal), any medication that she has been taking (nothing except pills), any relationship problem(no). On physical examination, the examiner said all normal except there is a paramedian scar on abdomen. Then I offered her different methods of contraception. Depo Provera (this is the injection form and need to inject every 3 months). I do not want to go to the clinic every 3 months, besides this is also hormone. Implanon (It is like a small stick which will be put under the skin of the arm) And last for about 3 years. (Mirena)_can last for 5 years suitable for those with heavy period No Male sterilization-Her husband does not want to have this. Explain its a permanent procedure, method, failure rate, possibility of Rejoining. It will not protect from STD, so u need to use condom. Reverse process is very difficult and involves complicated surgery (about 50% can be pregnant with high risk of ectopic pregnancy. Regarding the procedures, it is usually done with laparoscopy but in your case since you have the scar, I will refer you to the gynaecologist to discuss further about the procedure. RP asked me what about my period after sterilization. It will not change.

AMC Feedback: Request for sterilization(passed)

Paediatric
(1) 12 month old child, refuse to stand for 24 hours, presented to ED with her father. Task: take focused history; ask examination findings from the examiner, investigation and management.
History taking I asked the RP, from your notes I understand that your daughter refused to stand for 24 hours. Can you tell me more about it? RP said it happened all of a sudden and the child was crying excessively thats why he brought her to ED. I asked can she walk properly before. Yes. Has she been crying all the time or at particular time (like when u touched her limb). Yes when I was changing her clothes and holding her limbs, she cried a lot. Any trauma to the limbs. No. I forgot to ask whether there are any swellings in either of the limbs. Is she feverish? Yes, high fever for 48 hours. Any viral I illness before? Any fits, noisy breathing, discharge from ear & nose (No). What about water works (does the child cry when passing wee) and bowel motion? Normal. How is her general health so far?(healthy child)BIND Normal. Physical Examination General appearance (drowsy/alert) _child looks ill. Vitals (39C) ENT examination_ normal. Respiratory /CVS/Abdominal examination _normal. Lower limb examination what do u wants to know? I asked can the child stand(NO), bruise, deformity, swelling, temperature , tenderness (Examine joint above and below) there is a swelling , redness , tenderness in left knee joint and other finding are normal. Explanation and Management I explained to the RP: From history and examination, I am thinking a couple of possibilities. Septic arthritis , osteomyelitis and transient tenosinovitis. But most likely your child has a condition called septic arthritis that is inflammation of the joint due to infection through the blood. Your child need to be admitted to the hospital where she will be seen by ortho registrar. She will be put on antibiotics after taking blood sample to find out the causal organism, then joint x-ray, chest x-ray, other necessary investigations depending on the initial results RP asked: Is my daughter condition serious?

Yes, but with immediate treatment and proper care, she will be ok. How long does she need to take antibiotics? IV for 2 weeks followed by oral for 3 to 4 weeks depending on your daughter condition. Doctor will switch to the most appropriate drugs once blood result came back. Why did it happen to her? Bell rang AMC Feedback: Septic arthritis (passed)

(2) 4 year old girl went to ED yesterday due to acute attack of asthma followed by upper respiratory tract infection. Today, Father came to your GP clinic for follow-up. Task: take focused history, and explain the management plan to the father.
History taking From the notes I noted that the child got the acute attack of asthma yesterday? Can you tell me more about it? What did the doctors gave her? When was she diagnosed with asthma? (3). What treatment is he taking? Ventolin How do you give him ventolin ( 2 puffs with spacer ). I know how to give the medicine. ok I will check it later. Do you know how frequent you get the symptoms? 4 attacks within 8 months. What happens in between the attacks? Do you have symptoms between the attacks? Yes. How many days in the week does he have the symptoms? 3 to 4 days Does he have symptoms in the evening? How many nights does he have the symptom? 4 nights How is his sleep? (Can sleep well)Has he ever been admitted due to severe asthma? No. Just went to ED. How often does he get admitted? What do you think is the reason or what triggers her asthma? Viral infection , Flu, pets, smoking. Is he allergic to anything else? Explanation Then said, I will give your daughter ventolin only. Ask to avoid triggering factors and wash the spacer as well

If the child need ventolin more than 1 day a week Advise about activity and growth of child I gave Asthma action plan 3 copies Advice to stick the copy at the fridge Explain what to do when the condition become worse. Wherever you go Bring asthma action plan Always bring reliever, spacer

I failed this station because I did not add steroid which is very important. Although RP gave me hints to add steroid I could not respond. (this is my last station).

AMC Feedback: Chronic Asthma (failed)

(3) Mother of 4-year-old girl, who has separated from her husband, comes to your GP clinic as she noticed the rash in the vulva area of her daughter. She thinks that her Ex husband is abusing her daughter as the girl who has been visiting her father refuse to go back to her fathers house. Take relevant history, physical examination and manage the case.
History taking I asked the RP about her concern. She thinks that her ex husband is abusing her daughter. Why do you think like that? She said her daughter does not want to go back to her father place and she noticed the rash in the vulva area of her daughter. Ok, I understand your concern but before we come to this point, can I ask you a few questions. Yes. Start with the detail history of rash (site, onset, and colour), itchiness, vesicles, ulceration, bleeding, discharge, and trauma. Similar rash in any other part of the body. What about her water work and bowel motion? Normal. Have you noticed any abnormal behaviour like strange posture? No. How is her interaction with friends and with you? Fine. With whom is she living apart from you? (my boyfriend). Hows her relationship with your boyfriend? (Good) he is a nice person. Has she got any allergic history? SAD questions for all. Physical examination: GC alert and happy. Any bruise, rash, scar (all over the body) _No. External genitalia_rash (no bleeding, no ulcer, no discharge) Explanation and Management Based on history and examination, I could not find any evidence suggestive of sexual abuse. There could be other possibilities like (foreign body, allergy, and valvovaginitis). But I understand your concern for your daughter so I will refer her

to child protective service. They have a team where many people involve doctors, police, and social workers. They will examine your daughter thoroughly and take sample from your daughter private part. They will also interview you, your ex-husband and boyfriend. If you want I can offer you the support group. We will try our best to give you the best possible help to solve your problem. I will follow you up. Bell rang

AMC Feedback: Sexual Abuse(passed)

Psychiatry (1) 50-year-old woman, with long standing history of depression, has been admitted to the hospital for severe depression with psychotic features. She has been on antidepressant but not responding well to this medication. So, psychiatrist has decided to do ECT. Task: Explain to the daughter about the indication, the advantages and disavantages of ECT.
History taking Hi Ms . , I am Dr. . I understand that you are here to discuss about your mother condition. Yes. Ok, I will explain about your mother condition and the treatment she will need. Psychiatrist has decided to do ECT (electroconvulsive therapy). Have you ever heard about that? No, what was that? ECT is a short wave given with electrodes from which current passes to the brain that induces a seizure. The purpose is to change the level of chemical in the brain. RP asked me it sounds dangerous? I said it is safe and effective. It is given with general anaesthesia along with muscle relaxant. So the patient needs to fast before doing this procedure. She might feel confused, headache after waking up. Only in one condition like if there is any mass or recent haemorrhage, where we cant do this procedure. So we will do CT of head first to exclude this. RP: What do you mean my mother can have another condition? I said we just want to exclude whether is there anything in her brain that can increase the pressure in her brain. She seemed to be confused. Then I said this is part of the procedure. RP: Then she asked why they decided to give ECT to her mother. I said since your mother is not responding to medication, not eating nor drinking or not caring herself, she has psychotic features as well. So she can harm herself if we do not control her condition very well. Since we cannot wait for 4-6 weeks for the antidepressant drugs to kick in, this is the most effective way of treating her depression.

RP: Why did not they give ECT if the response is good? Why did doctors treat her with antidepressant without doing ECT previously? We decide to give only to those who have indication like severe depression and suicidal ideation(like in your mother case). Otherwise, we only use antidepressant to control mild to moderate depression.

RP : Will my mother condition be cured after giving ECT? We uusually give 6-9 sessions of treatment to patient. But, sometimes people respond after 3-4 sessions. RP: What do you mean by respond? I mean if her condition improved after giving ECT, we will consider continuing or not. Number of treatment given will be decided by psychiatrist. RP: Are you sure that my mother condition will be completely cured after receiving ECT? I said the condition will be improved but not cured so she will need to take antidepressant. It depends on individual patient, if u have concerned I can arrange meeting with psychiatric to clear any doubt. I could not convince to RP and the bell rang.
AMC Feedback: ECT (failed)

(2) 18-year-old with DM type1 has been in hospital because of DKA. Normal weight. She has a history of binge eating, use laxative and history of self induced vomiting after binge Eating. Now she is ready to be discharged. You are a HMO of that hospital. Task: Explain her about the eating disorder she has and counsel her about the management.
This is exactly same recall with opening statement Are u going to tell my GP about this I ensure her about the confidentiality. Explained her everything. But when I came out of the room I realised that I forgot to mention the name of this condition (bulimia nervosa) which is one of the task.

AMC Feedback: Bulimia nervosa (failed)

Medicine & Surgery


(1) 60-year -old man went to the hospital where stressed ECG test was done and it was found to be positive for ischemia showing ST segment changes (probably anterolateral segment). The patient records were given. He has

history of smoking for 20 years. Blood pressure is 130/80 mmHg, cholesterol is 6 mole/l. Now he is on anginine. His BMI >35. Task: Explain to the patient, the results.
I started this station with explanation of the result Mr from your notes, I understand that you are here to discuss about the test results. Am I correct? Yes. Your stress ECG result shows that you have a condition called angina pectoris. It is a condition where people present with chest pain due to shortage of oxygen to heart muscle. It occurs due to deposition of fat like substance in the coronary vessels resulting in narrowing of blood vessels that supply to the heart. It can be due to several factors, so we need to control risk factors first. Lifestyle modification: The first one is alcohol since you do not drink alcohol which is good. Second, blood pressure is also within normal limit which is again very good. But your BMI is about 35 which fall into the category of obesity. So I will refer you to the dietician first to work out on a diet plan and I would like to suggest you to do exercise like brisk walking but not at the moment. For that I would give you the contact of the trainer, they will assess you and give you the exercise program within your limit. Aim for body weight is between BMI 20-25. Hopefully we will try to achieve that level gradually with diet and exercise. If not, then we will consider using medication. I also found that you have been smoking . . . cigarettes/day for over 20 years. Since cigarette smoking is one of the risk factor for your condition. Would you like to consider stopping smoking? Ok, I will arrange consultation to quit smoking. Cholesterol level is 6mmol/l (slightly high), but I need to order lipid profile to check other lipid and will control the level with diet. Then, if failed, we may start you on certain medication which lower cholesterol called statins. Do you have any concern? Is my condition serious? Not at this moment if we control all these risk factors aggressively, we can prevent serious outcome. So, I will refer you to the cardiologist, he will do necessary investigation and depending on the result the cardiologist may decide next step. Then RP asked me can I have sex, yes but once your condition has stabilized. But I would like to advice not to use drug like Viagra. Bell rang. Could not say anything more. Feedback: Angina Pectoris(passed)

(2) 50-year-old man comes to your GP clinic that has been referred to you by another doctor with the complaint of tiredness. He is non alcoholic. He has inserted pacemaker for bradycardia, abnormal LFTs (ALT -raised, Alkaline Phosphate-slightly increase), Bilirubin-normal, Viral serology-Negative, USG showed slightly enlarged liver, small gall stones, Task: Ask further investigation, counsel the patient depending on the results.
From this referral notes, I understand that your previous GP ordered some blood tests because you complained of tiredness. And the results showed that your liver enzymes are increased, but the viruses that can attack the liver and causing raised liver enzymes are not found. The USG showed small gall stones and slightly enlarged liver. You have inserted pacemaker for heart problem. Since you have raised liver enzymes without drinking alcohol and not having viruses in the body, I am thinking about the condition called haemochromatosis. But I still need to confirmed this by ordering iron study (at that time examiner gave me the result), yes it showed serum ferritin and transferring saturation is increased, which means storage of iron in the body is high. But I need to order HFE gene study (examiner gave me another card) which showed C282Y is positive. So I explained to the patient that my suspicion of haemochromatosis is confirmed by this test. I asked for blood sugar level, first examiner said not available then said 8mmol/l (I guess no other tests are available). I asked RP do you have any idea about this condition (haemochromatosis). The Rp said NO. It is a genetic condition where there is a defect in the gene which helps to regulate the amount of iron absorbed from food. Because of this defect the body absorbs too much iron which deposits in different organs if not detected and treated early, skin (pigmentation), liver (cirrhosis), heart (rhythm problem), pancreas (diabetes_DM), testis (atrophy, loss of sexual drive). That's why people of this condition usually complain of fatigue, lack of energy, abdominal pain, loss of sex drive, and heart problems. Treatment is simple, just to remove 500ml of blood every week until iron level goes back to normal Treatment cannot cure the conditions associated with hemochromatosis, but it will help most of them improve, which leads to a normal life expectancy. However, if cirrhosis (scarring of the liver), has developed, the future outcome might not be good. I will refer you to the specialists, hepatologist; he/she will do biopsy (taking the tissue sample from the liver). Do u have children? He said no.

Do u have siblings? Yes I said all siblings (first degree relative) should do blood tests to see if they have the disease or are carriers. Gall stones are just incidentally found. For this I will refer you to the surgeon with whom u can discuss about the treatment option. Examiner asked do u think the raised liver enzyme could be due to gall stone. I said no.

AMC Feedback: Abnormal Liver function Test(passed)

(3) 22- year- old young man came with severe headache. You are HMO at the ED of the hospital. Task: Take history, ask physical examination from the examiner and explain diagnosis to the patient.
DDx of headache: migraine, tension headache, tumor, haemorrhage (trauma), cervical spondylosis, sinusitis. History taking I asked the patient Are u comfortable to answer my question or do u want me to relieve your pain first. RP said its ok. Then I started with pain questions where, onset, severity, radiation aggravating or relieving factors. [Pain around the head for 2 days, severe, not relieved with pain killer, no nausea &vomiting, no fits but sensitive to light]. Any trauma recently (No). Any pain in neck? (NO). Any problems with the sinuses. Any weakness in any part of the body. Any similar attack before? All No. Any contact with sick patient recently. NO Physical Examination GC: conscious & ill, No rashes. Fundoscopy (cant see properly due to flashing light, need to dim the light?), Neck stiffness (positive), Kernig sign (Negative). Neurological examination (Tone, power, reflexes) - All normal Urine dipstick (normal) Examiner asked me to talk to the patient about the diagnosis. Explanation From history and examination results, the most likely condition you have is what we called meningitis (inflammation of the covering of the brain), but I still need to rule out some lesion in the brain (tumor). He asked me what else; I forgot to mention (haemorrhage). You need to be admitted to the hospital. Is my condition serious? Yes. I thought that it is just normal headache, s o I was staying at home. At the hospital, you will be seen by neurologist and will order CT scan, (to rule out the condition that can increase intracranial pressure), blood culture, FBC, Blood glucose, ESR, CRP. Then, they will consider to do LP if CT normal. You will be treated according to the results.

AMC Feedback: Headache (acute) passed

(4) 50 years old, gardener, complains of long standing back pain and groin pain. Take history; ask for examination findings and discuss your management to your patient.
I started with pain question: Pain around groin area, deep seated pain, severity 6/10 , no radiation, no pins and needles sensation. Back pain, for a long time, diagnosed with lumbar spondylosis. Relieved with physiotherapy now, back pain is not that much giving him trouble. Groin pain which occurred after lifting something. Pain aggravated by climbing stairs, not with walking or sitting. Not relieved with pain killer. Besides No history of trauma. He has no other medical problems. Examination findings will be provided which u asked for. I asked BMI (as u see), posture, gait, movement, power, sensory, special tests. Results were limitation of Abduction, Adduction, IR, ER, Flexion, but Extension-normal. Tenderness at the vertebral column (not specific) No fix flexion deformity, Straight leg raising -normal. Sensory and reflexes were normal. I explained to the RP that from history and examination findings, I think you have a condition called osteoarthritis, but at the same time I need to rule out hip dislocation & fracture (which are less likely), and sciatica (less likely). I will order x ray & see is there any arthritis changes or fracture or dislocation) If not could be nerve problem. So if necessary I will refer, bell rang. Could not say anymore.

AMC Feedback: Hip Pain (Passed)

(5) A 50 years old female was diagnosed with ca caecum about a few months back and surgical procedure was done (entero-enterostomy). Few days back, she presented with intestinal obstruction when she was readmitted to the hospital where operation was done again to relieve obstruction. But there were some cancer cells left in the pelvic. Now she had severe pain in her lower back and lower tummy region which was not relieved by pain killer. Physical examination showed tumour has spread within the pelvic and is now in incurable stage. Task: Counsel the patient.

Breaking bad news: As usual

AMC Feedback: Carcinoma of colon (recurrent , advanced) passed

(6) 70 year-old lady with L4 compression fracture and back pain with t score -3 (DEXA scan), come to your GP clinic for result. Take history, explain the nature of the result and counsel her regarding management.
I asked the RP about the risk factors (body weight, cigarettes smoking, alcohol drinking, exercise, calcium in her diet, taking cortisone and HRT, family history). Her body weight is normal. She has had menopause for 15 years but not on any hormonal therapy, or on any medication such as prednisolone. She has no history of hypertension and diabetes mellitus, thyroid and renal disease. She does not smoke but consumes 2-3 drinks of alcohol a day. The patient doesn't like milk, dairy products and doesn't do any exercises. Last Pap smear was done _ years back which was normal and mammogram done 6 months back that was normal too. Family history I explained to her about the results. DXA scan which can predict an increased risk of osteoporosis and fracture showed t score -3 which means you have a condition called osteoporosis. Do you have any idea about this? It is a condition leading to thinning of bones with reduction in bone mass due to depletion of calcium and bone protein so that they become weak and brittle and prone to fractures. It is found mainly in middle-aged and elderly women, after the menopause where the body (ovaries) stop producing female hormone called oestrogen which is important for forming bone mass. Another reason is lack of Ca in the diet and lack of activity. What I would like to do now is I will give you the pain killer to relieve the pain first, Take regular weight-bearing exercise such as walking (e. g. Brisk walking for 30 minutes 4 times a week). RP asked me do I need to do exercise now. No I will refer you to specialist for fracture. You need to take rest till pain subsides. Then you will be put on bisphosphonates and calcium supplements. I advice her to limit alcohol drinking within safe level. Encourage her to take dairy products and provide her pamphlet containing the lists of products and the amount she need to take. Any concern? RP asked me can there be other cause for this fracture. Then I said yes I will refer you to the specialist to rule out a condition called multiple myeloma. AMC Feedback: Osteoporosis (passed)

(7) 55 year old lady presented to GP with the complaint of swelling and pain around lower part of right leg. Mother and sister have DVT. She is taking HRT. Task: Take further history, ask for investigations and explain immediate and future management.
AMC Feedback: DVT (passed)

(8) 60-year-old man, chronic smoker, recently becoming more and more SOB on walking up hills. Task: Do complete respiratory examination within 6 minutes and present your findings to the examiner.
AMC Feedback: Respiratory Examination (passed)

Sydney 9th March 2011, morning session.


1. A middle age woman has major depression with psychosis and suicidal idea

not responding to antipsychotics and antidepressants. She is in hospital now and the psychiatric team decided to give her ECT. Her daughter is concerned about procedure and sees you in GP office. Tasks: explain the procedure, its indications, advantages and disadvantages, address the concerns of the daughter.

I explained ECT-the use of electricity to correct the chemical imbalances that cause your mothers symptoms. It is performed under general anaesthetics. We apply the electrodes on the head, then deliver electrical activity that cause convulsion and correct the problem.

It is indicated when the patient has severe mental illness not responding to medication such as major depression, acute psychosis and is at risk of harming themselves or other people, and it is the case of your mother. Also indicated when patients cannot take medications because of other medical problems. Advantage: quick method to help the patient.

Disadvantages: temporary memory loss, which may be persistent, risks of anaesthetic drug reaction. Questions from the role player: It looks terrible on movies, is it sore? Yes, it looks more terrible than it is. We work together with psychiatric team and anaesthetic team; make sure that your mother is not suffering. She will be put to sleep and when she wakes up, everything is done. She would not feel sore when she has the procedure. Will my mother loose her memory and cannot recognise her children? She may have temporary loss of memory but will recover with time. Only a small chance that memory loss may be persistent. We need to consider the benefit of treatment and its side effects.

I did not know what else to discuss as I covered these points within 3-4 minutes. I asked the role player several times whether she has any other concerns. She said no. I failed this station and did not know why.

Feedback: ECT- procedure, indications and effects

2. A young lady with type 1 diabetes admitted to hospital because of DKA. A ward nurse said she has confessed to skipping insulin and inducing vomiting after having heavy meal. Psychiatric review confirmed that she has bulimia nervosa. Her weight is normal. She is now stable and about to be discharge. Task: take relevant history, explain the diagnosis and management.

I addressed confidential issue before taking history. She has supported family; she is working, no financial problems. She has had a good relationship with her partner but was upset when he gave nice compliments to the dress of another girl who looked much skinnier than her. Since then, she has tried to lose more weight by diet because she thinks that she is too fat. Sometimes she eats excessively and induces vomiting because of feeling guilty. Sometimes skipped insulin and thinks it works. No excessive exercise, no depressed mood, sleeping normal, appetite normal, no delusion or hallucination, no suicidal idea. Periods have been regular.

I said your weight now is normal and you do not need to lose more weight. I understand you are very concerned about your weight and that concern affects your health. She asked why? Because if you try to induce vomiting there is an increased risk of tooth decay, unstable blood sugar levels, and electrolyte imbalance that can lead to death. Skipping insulin can also worsen diabetes control. Id like to refer you to a counsellor who can help you to cope with your concern; also refer you to a dietician who can give advice on good diet. She was happy with my advice but at this stage she said I dont like to see psychiatrist. I said if you dont want to see a psychiatrist, thats OK but at the stage that you cannot cope with the anxiety about your weight, your diabetes can not be controlled well, the psychiatrist can give you medications to reduce the anxiety about your weight. She looked happy but I failed this station unexpectedly.

Feedback: Bulimia Nervosa

3. A 3 year old girl was discharged from hospital 2 days ago because of asthma attack. Her father comes to see you today in GP practice for following up. Tasks: take relevant history, write asthma action plan and explain to the patient. On history, the patient has had asthma since she was 1.5 years and has been treated with ventolin puffer when she has symptoms. At the moment, she has had regular day and night symptoms, including cough, SOB, wheezing. She also has symptoms when playing. There were no known allergies, eczema, no carpets, no pets, no smoker at home. On discharge from hospital, she still has 3 days of prednisolone tablets to go. There was a stack of asthma action plan forms on the table. I took one after asking the examiner and filled it up. I added daily corticosteroid inhaler apart from ventolin and explained what to do when well, unwell, get worse, asthma attack. Time ran out when I was explaining 444. The examiner was very nice, gave me her empathic smile when I had difficulty explaining 444 strategy in the last seconds. Feedback: Asthma (paed) 4. A mother of a 4 year old girl came to see you in GP practice, complaining that her daughter does not want to go to her ex-husband. She noticed some redness in the vulva and suspected sexual abuse. Task: Take relevant history, ask physical examination and manage the case

The mother was very angry. The baby refused to go to her fathers house for couples of weeks after coming back from her fathers house. She did not tell what has happened. The babys parent separated one year ago and she live with her mom the new partner. Her father at the moment also lives with his new partner. Her development has been normal. On examination, the baby was playful, no positive findings apart from mild redness and swelling in the vulva area. The babys mother asked: do you think that her father abused her? I said that could be one of the possible causes. Other possibility is inflammation of the skin, a common condition at this age. However, I will refer your baby to child protection unit, there are people that can examine your baby, take swab from the vulva to confirm the diagnosis. They will also come to interview those related to caring for your baby such as you, your partner, your ex-husband and his partner, and me as well. I expressed empathy to her concern with reassurance of being supportive to her. I finished this station early after 3-4 minutes- nothing else to say Feedback: Sexual abuse (paed).

5. A father of 2 year old girl came to ED because his daughter cries when he tries to pick her up. Tasks: take relevant history, ask physical examination findings from examiner and explain differential diagnosis and management to the patients father.

On history, the baby has been well until now, normal development. Today she refused to walk. She is feverish. No other symptoms.

On examination, she is 39oC, no dehydration, no blotchy cyanosis. Hip examination normal, knee examination: red and swollen left knee, limited range of movements.

I explained the diagnosis of septic arthritis, with osteomyelitis as the differential diagnosis.

For management: this is an emergency condition, baby needs hospital admission, we will put a canula for her, take blood for investigations, call orthopaedic registrar, arrange X-ray of the knee. Treatments involve

antibiotics for at least 3 weeks after taking joint fluid for investigations; she may need operation to drain the knee joint. Both examiner and role player were happy. Feedback: Septic arthritis (Paed) 6. A 35 year old woman comes to see you to ask about sterilization. Tasks: take relevant history, ask for physical examination, give diagnosis (?), explain the procedure and other methods for contraception. The patient has had 3 children, no significant obstetrics and gynaecology history. She has had laparotomy for appendix abscess long time ago, very keen on sterilisation and does not want to discuss other options.

Physical examination revealed a scar on the abdomen due to previous operation, other examinations was normal.

I explained about tubal ligation procedure, the risks of anaesthetics. Refer her to gynaecologist to consider whether keyhole surgery is suitable because she has the risk of adhesion due to previous operation.

She asked about the failure rate- I said 1 in 200. I finished early. Feedback: Request for sterilisation (O&G)

7. A young woman comes to ED because of severe abdominal pain. Tasks: take relevant history, ask for physical examination, give diagnosis, differential diagnosis and management.

When I entered the room, the patient looked in agony because of pain. She held her hands on the right iliac fossa; gave history of pain with sudden onset, felt a little bit faint, pain was constant, no radiation, severity 8/10, nothing could help to relieve the pain, no nausea/vomiting, no problems with bowel motions or water work. She had stable partner, LMP 7 weeks ago, no contraceptive methods, no previous pregnancy or STD, recent PAP smear

normal, no pain/bleeding on intercourse, no vaginal discharge or bleeding. She was otherwise healthy.

On examination, vital signs normal, suprapubic tenderness, McBurney, psoas signs, Rovsing sign negative, no loin tenderness, Speculum examination: cervix close, no discharge. Bimanual examination: uterus size normal, cervical excitation negative, no adnexal mass or tenderness. Urinary pregnancy test positive.

I said most likely you have ectopic pregnancy, cannot rule out appendicitis, threatened abortion. You needs hospital admission. Well put canula, take blood for serum B-hcg , preoperative assessments, blood group and cross match, transvaginal US, call gynaecology registrar, prepare operating theatre, may need operation. There were no questions from the role player and examiner.

Feedback:Possible ectopic pregnancy (O&G) 8. A young woman at 10 week gestational age comes to see you in GP setting. She is concerned about having a baby with Down syndrome as recently her sister just had a newborn baby diagnosed with Down syndrome. Task : counselling the patient The woman was very anxious about her pregnancy. She said- I dont want to have a Down baby, is there any way to detect it early so that I will not continue the pregnancy. I said yes, there are some options for you to detect Down syndrome during pregnancy. Because you are under 11 weeks of pregnancy we can arrange Ultrasound at 11th week to see the presence of increased thickness of the space behind the babys neck. This combines with some other blood tests such as B-Hcg, pregnancy associated protein A taken at the same time can detect 90% Down syndrome. The other option is doing blood tests at 16th week including B HCG. Estradiol, alpha fetal protein. These tests are used to screen for Down syndrome. If the tests suggest that you may have Down baby, we need to confirm diagnosis by either taking a sample from placenta at 11th week or taking fluid surrounding baby.. She asked: Is there any risk for those procedures? The risk of abortion is less than 0.5% for amniocentesis and less than 1% for CVS.

Are the tests accurate to detect Down syndrome? _yes, most likely but there are no test can give 100% accuracy to diagnose Down syndrome. (I knew my answer was risky but anyway I passed this station). If the tests confirmed that I have Down baby and I dont want to continue pregnancy- can I terminate it? By which way?- If it is the case, I will refer you to the gynaecologist to discuss more about termination. Generally under 16 weeks, we can do suction; above 16 week, we induce labor.

Feedback: Explantion of fetal abnormality screen.(O&G)

9. GP setting, a middle man has had chest pain, SOB on exertion. On examination, he has BMI 35. Cholesterol 6.0, no hyperglycaemia, Stress test showed ST depression Task- explain about investigations, diagnosis and management to patient.

I explained about the investigations; referred him to a cardiologist to consider angiography and other procedures such as stenting, bypass surgery. I spent a lot of time to explain about cardiovascular risk factors (smoking, overweight, hyperlipidemia) and did not have time to start him on statin, aspirin, GTN. while waiting for the specialist consultation. I failed this station. Feedback: Angina pectoris.

10. A middle age female patient came to see you in GP Office complaining of left hip pain. Tasks: take relevant history; ask physical examination from examiner, give provisional diagnosis and further investigations.

In this station, Dr Wenzel was the examiner. The patient has had hip pain recently (I cannot remember for how long). Pain felt in the lateral part of the groin, no radiation, worsens when walking, reduces with rest, develops gradually, no tingling, numbness, weakness of the leg, no previous injury to the hip, no morning stiffness, no fever, and no weight loss. No significant illness in the past except arthritis on fingers (PIP and DIP joints). No other joint pain. Not on any medication; non smoker.

On examination: left hip joint has pain on movement, limited range of internal rotation and abduction. No abnormality on neurological examination.

My provisional diagnosis was osteoarthritis. I explained it to the patient and arranged X-ray of the hips.

There were no questions from the role player and examiner, who gave me a big smile when I finished.

Feedback: Hip pain.

11. A 70 year old woman who has developed severe lower back pain and XRay shows compression of lumbar spine (L4L5). DEXA bone scan -3SD. Tasks: take relevant history, explained to patient about the diagnosis, provide management.

On history, the patient still has pain, no previous illnesses, non smoker, not on any medication, no weight loss, no fever, no tiredness recently. Systemic reviews of symptoms were all normal. The patient does not take any dairy products, no exercise, not much outdoor activities. Menopause more than 10 years; last Paps smear 2 years ago normal. Mother had osteoporosis at the age of 80s.

I explained about the diagnosis, offered further investigations including bone scan, blood tests, urine tests to rule out multiple myeloma and other malignancies.

For management, I gave her pain killers, biphosphonate and mentioned its side effects, how to use, advice on dietary, referral to dietician for advice on good sources of Calcium rich foods, exercise to strengthen the muscles, referral to occupational therapist to make sure safe home environment to prevent falls. The role player nodded her head but said nothing. Feedback:Osteoporosis.

12. A middle age woman developed lower leg pain and swelling after a long trip comes to see you in GP office. She has been on HRT. Her mother and sister has had DVT. On examination her leg is swollen and red with pitting oedema. Tasks: take relevant history, ask examiner for investigations, explain diagnosis and management.

On history, pain developed suddenly, she had been fit and healthy, no diabetes, hypertension, no smoking, no previous leg injury, not on other medication apart from HRT. Her sister had DVT after having delivered her baby; her mother had DVT because of . (I cannot remember now, please check other recalls).

For investigations, I asked for Doppler US, the examiner nodded his head and said positive. ABI normal range.

I explained the diagnosis of DVT and the associated risks, recommended hospitalisation in which she will have bed rest, blood tests for coagulation profile, thrombophillia screening, introduction of Heparin and Warfarin at the same time and relevant daily blood tests, stop heparin when INR reaches therapeutic range, stop HRT or use the other form because of high risk.

Questions from the role player: Duration of using warfarin- Depends on the thrombophillia test results , 36 months or lifelong. Therapeutic range of warfarin [2-3]

Feedback: DVT.

13. A middle age man comes because of SOB developed gradually. He has been a heavy smoker, about 2 packs a day. Task: perform comprehensive respiratory examination and give diagnosis.

This was a real patient. I performed respiratory examination according to Talley and OConnor. He had no Horners syndrome, no cervical lymphadenopathy, positive finding included expiratory wheezing on over the

lung fields. When I was listening to the chest, the examiner said you have 30 seconds for examination. I quickly said- Id like to do PEFR test. The examiner said do it! The instrument was provided on the table with disposable mouthpiece. The patient performed it correctly at the second go. The examiner then stopped me and said other repeating gave consistent results, and then he told me to interpret the result which indicated 120 ml. I said-there is evidence of airway obstruction and Id like to perform the PEFR test after bronchodilator to confirm COPD. He asked me what is the other bedside test? I said spirometry. He nodded his head with satisfaction. The bell rang. Feedback:Respiratory examination.
14. A long stem with complicated details which indicated colon cancer previously

operated but now having peritoneal cavity spread, free fluid inside the abdomen, no liver metastasis. The patient was a woman on middle age. Tasks: explain the diagnosis and management.

I gave breaking bad news steps by steps. She took it quietly and asked what the available treatments for me were. I said most likely chemotherapy and/or radiotherapy. Treatments mainly to reduce symptoms that she may have such as bowel obstruction with colostomy; pain with pain management we work together in team including oncologist, surgeon, pain management specialist, community nurse, social worker to help you. You do not need to stay in hospital all the time, just 1-2 days if you have chemotherapy/radiotherapy to follow up the side effects. She asked: How long will I live..? It is different among individuals, many people with this condition live longer than they have expected. We will help you to enjoy activities as much as you can.

I also suggested cancer screening for her children. Feedback: Carcinoma of colon (recurrent, advanced)

15. A young patient with headache developed 24 hrs ago. She is now in ED. Tasks: take relevant history, ask for physical examination, and explain possible diagnosis and management.

On history, the patient developed headache for one day, on over the head with soreness in the neck. Pain was constant with 8/10 severity, no radiation, feeling hot, no nausea or vomiting, no visual disturbances. Systemic reviews were all negative.

Physical examination: Temp 39oC, BP normal, neck stiffness positive, fundoscopy not available. Other examinations were normal.

I explained to the patient the possible diagnosis of meningitis, could be also subarachnoid haemorrhage, brain tumor. She needs hospital admission for having CT scan of the brain to exclude increased intracranial pressure, lumbar puncture, blood tests to confirm diagnosis. Treatments include antibiotics and symptoms management. The patient asked- what is CT scan, lumbar puncture?

Questions from examiner: What are you looking for in CT scan?- Brain tumor or evidence of increased intracranial pressure. CT scan and lumbar puncture - Which one you give first? Why?- CT scan first to exclude increased intracranial pressure, because of the associated risk of coning of the brain. What is the other test apart from CT scan to check ICP? Fundoscopy. What would you expect from lumbar puncture?- Biochemistry change including level of glucose, protein; numbers of white blood cells, culture for the responsible bacteria.

The examiner was happy with my answers. Feedback:Headache (acute).

16. You are in GP practice. A patient just comes to your area with referral letter from his previous GP. From the stem and the letter, you can see he has had pace maker, persistent abnormal liver function tests (increased AST, ALT, GGT), hepatitis A,B,C serology were all negative, abdominal US shows abnormal liver consistent with cirrhotic change.

The patient has had healthy lifestyle, no alcohol consumption. Tasks: explain the tests to the patient; possible diagnosis, order other tests to confirm diagnosis and management

I explained about the test, explained the provisional diagnosis as haemochromatosis and order Iron studies, genetic test, liver biopsy. The examiner gave me result cards only for the tests that I have requested. Iron studies show transferrin saturation 85%, increased ferritin. Genetic tests reveal C282Y positive, C63D negative Liver biopsy shows fibrotic change grade 2. I said you have some level of liver scarring as the result of haemochromatosis. We will need to take blood every week to reduce iron load, and the gastroenterologist will consider liver transplantation when your liver is not working properly. Because this is a genetic condition, your first relatives will need screening tests.

The examiner looked happy. Feedback: Abnormal liver function tests.

AMC Clinical Exam Recall 2nd April 2011


Station 1 Total cholesterol 7.3, HDL- normal, but lowish, LDL and TG high BP: 140/85, unhealthy diet, Father CABG, mother DM, non-smoker, social drinker Task: History, counsel regarding hyperlipidaemia Candidate 1 (Adelaide) Status: Passed Just CVS risk assessment (modifiable and non-modifiable) station and counselling; All positive risks were given as in the stem, social drinker, non-smoker, no exercise; just the RP wants to know more about healthy diet and reassurance + referral to medics and start on lipid lowering agent Candidate 2 (Brisbane) Status: Passed Same as above!

AMC Feedback: Hyperlipidemia Station 2 T1DM, GP, 7 yo male, 1 wk h/o lathergic, noturia, urine sugar +, ketone+, BSL 23, but, PE completely normal and the child is well at this stage. Task: Explain about the result and management. (no more history or PE) Candidate 1 (Adelaide) Status: Failed; dont know why Modified book case Newly diagnosed T1DM, need to refer to hospital, r/v by paeds, do some fasting blood: fasting BSL, Lipid, Insulin Assay, Explain about DM, RP asked me what dose ketone stand for and the significane (ketone means dehydration in your son, and bcoz sugar is osmotically active and draws water, hes got increased frequency and amt of urine and also there might be some electrolyte imbalance causing him weak and lathergic and thats why he needs to go to the hospital and assessed by paeds, but, not definitely necessitate admission since hes physically well at this stage.) If confirmed, he needs to be closely monitored, BSL 4 times a day, insulin injection for life long (mum and dad are primary carers of the boy), but, dont worry too much, ppl live with this condition and with proper monitoring and control, he can lead normal life, and he can also play sports and no specifically restriction of activities, also warn about hyper and hypo-glycemia and glucagon, blah, blah, blah, the RP wants to know all the Q in book case and show interest in Ketone and hypoglycaemia and glucagon injection. Candidate 2 (Brisbane) Status: Passed Quite similar but I explain more about DKA and to exclude that the boy needs to go to hospital and reviewed by specialist. The patients questions are quite similar to the book case. AMC Feedback: Newly diagnosed child DM Station 3 - Rest Station 4 Day 5 postop DDx, PE 50+ lady, day 5 post-op for total knee joint replacement surgery, had 2 days of heparin subcut 5000 BD and prophylactic ABx post op. Now complain of SOB, sweaty, anxious. Task: History for 3 mins, asked PE findings from examiner (only give what you asked for), Invs and Dx (not management) Candidate 1 (Adelaide) Status: Passed When I went into the room theres a lady lying on the bed in hospital gown, the examiner greeted me and I started to ask whether my patient is vitally stable or not and he said shes stable. Day 5 post-op, not much walking post-op bcoz of pain Sudden SOB, sweaty, anxious for last 2 hours, asso: chest pain-> central, no radiation, no N+V, no recent H&M, no cough, no calf pain. 1st time having this kind of problem PMHx: otherwise well apart from knee arthritis. NKDA, no meds apart from aspirin, no significant FHx.

Then, I asked for PE findings. The examiner looked strict at first sight and asked me what I would like to know. So, I asked for general appearance which is anxious and sweaty. Vitals, Temp- afebrile, PR- 110/min, BP slightly lowish, SaO2 89% on RA (then, I asked the examiner whether my patient has the IV cannula insitu since she is not vitally stable and I would like to start her on oxygen 2L via NP and start to give her IV Fluid (then, I asked the patient again that she doesnt have any heart failure and the patient said no). Then, the examiner, just gave me the rest of the physical examination findings which are all normal (normal wound site, no calf tenderness, no signs of chest infection) Then, he asked me my DDx. I told him 1. AMI 2. PE 3. Septicemia, then, he asked me what are you going to do next and asked me to tell the patient? So, I explained my patient that your SOB and Chest pain worries me and I would like to take ECG, CXR and I would like to take some blood from you and run the blood for Full blood count to look for trace of infection, baseline renal and liver function test, and also, coagulation studies including D-Dimer since Im afraid you might also have the condition where theres a blood clot formation in your leg veins and dislodged into the blood stream and obstruct the major artery supplying the lungs and the patient looks a little bit worried. And so, I told the patient, not to worry too much and in the meantime, id like to give you some O2 to relieve your SOB and start on fluid. Then, the examiner, interrupts me that what was your first DDx (stupid me forgetting troponin even until that time :P), So, I told him that my first DDx was AMI and then, he asked me what investigation would you like to do to confirm that Dx. Then, I know that I forgot Cardiac enzyme and I wanted to laugh at myself and told him I would also like to run Troponin. Then, he asked me the normal value for troponin. I answered < 0.04 and he said the result came back like that and so, what is your first DDX now. So, I told him that now, PE is my first Differential so, he asked me what would you like to do to confirm PE and I told him Id like to do ECG which will show Sinus tachycardia and S1Q3T3. He smiled and then, he gave me the ECG and asked me to interpret that. So, I presented the ECG (this is the ECG of Ms. X, age 50 yr, HR around 120/min, sinus rhythm, tachycardia, and I can appreciated S1Q3T3, but, I cannot find any ST or T segment changes. Then, he asked me what else invs? D-dimer which will be raised by thousands, but, to confirm that I need to do CTPA. He smiled and then, the bell rang. I felt very good after that station bcoz the examiner, the observer and also the RP looked strict at first, but, actually, the examiner was very helpful and said thank you very much and also, the RP gave me a very big smile afterwards Candidate 2 (Brisbane) Status: Passed Same as above! AMC Feedback: Chest pain and dyspnoea (acute) Station 5 30 yo, P2, 4 mo post partum, came to your GP for wetting problem, Ht and weight was given. Task: History for 3 mins, PE from examiner (will only give what you ask for), Dx and mgt to the patient. Candidate 1 (Adelaide) Status: Passed I went into the room and congratulated the RP for the baby and asked if everythings well with her and baby. She said apart from the wetting problem, shes coping well and the babys fine. Q. How long youve been having this problem? Since before this baby born, for about a year now, gradually worsening and concerns bcoz shes still young.

Q. Is that worsened when you cough or strain? She said yes, esp;ly when coughing and sneezing Q. When you wet yourself, is the amount a lot or just a little bit? Just a little bit Q. Do you need to rush to the toilet when you need to go? No Q. Do you have any other problem with the waterworks, like burning sensation, stingy pain or fever? No Q. How about the bowel? Fine Q. any N or V? No Q. Can I ask you some sensitive question? Yes Q. Ask about 5P and all normal. Q. How about your babies and deliveries? All normal vaginal delivery and both of them >=4 kg, no prolonged labour or prolonged hospital stay. Q. Any past surg or gynae procedures? No Q. Any past illness, sugar, blood pressure, lipid problem? No Q. SAD? No NKDA, not on regular meds PE: Normal general exam and system r/v, BMI 30 With the patient consent, PV exam -> Demonstrable stress incontinence + and also, slight anterior vaginal wall prolapsed Bed side UA, BSL normal Then, I explained to the role player that shes having a condition what we called stress incontinence (incontinence mean that you cannot control your waterworks so well and stress mean when there is increase stress or pressure in her tummy like coughing or sneezing). She seems to have understanding with my explanation and I told her that the most likely cause of her problem will be due to the big babies she had and overweight might also contribute to that. Then, I told her that I will refer her to the gynaecologist and she will be reassessed again and treat accordingly. The possible treatment option will be starting with general measures (its good that you dont smoke and keep that good habit, exercise, including the pelvic floor exercise) and also, therell be option like vaginal pessaries, and also surgical option. I asked her whether shes okay if she needs surgery and she asked me back what do I think? So, I told her that I suggest we will start with conservation measures and if not we can consider surgery and I try to tell her the 4 surgical procedures for GSI, but, the bell rang at that time and then, I realized that I didnt specifically told her to reduce weight. Candidate 2 (Brisbane) Status: Passed Same as above! AMC Feedback: Urinary Incontinence (Passed) Station 6 50 year old lady you are seeing in hospital OPD with complain of 2 wk ongoing SOB (the Q is that short) Task: History for 3 mins, DDx, PE from exmr, Mgt plan Candidate 1 (Adelaide) Status: Passed When I entered into the room, the RP said the examiner hasnt returned yet, and asked me to start with her. So, I introduced myself and asked history from her. Shes feeling breathless for about a month, but, worsen in 2 weeks and said that now even with daily chores and walking, shes SOB. But, no SOB at rest, no orthopnoea or PND. No chest pain, dry cough as usual

(smoker), no phlegm, no recent LOW or LOA, no stress, system r/v normal. Smoker 20/day for about 15 years. The examiner came back after about 2 mins, quietly sitting and then, after ive finished my history taking, he pointed me the next task and so, I told him my DDx: Heart Failure, COPD, LRTI and then, he gave me the PE result sheet. Vitals- normal, CVS: normal. Lungs, dullness on percussion on right lower zone, reduced AE, Vocal fremitus and resonance Reduced. And then, he pointed me the next task and pointed at the RP. So, I started to explain the RP that from my Hx and PE, you are having a condition what we called Pleural Effusion, in which fluid is trapped between covering of the lungs, which might be result of infection, inflammation somewhere or nasty growth itself. So, I need to refer you to the medical specialist and do CXR first to confirm the Dx and then, do the aspiration of the fluid to examine under microscopy to make sure its origin. Then, the RP asked me you mean nasty growth is that cancer? and she looked worried. So, I told her that it is not yet confirm as the cause, but, it is one of the possibility and we do need to exclude this by doing the aspiration, which is also kind of therapeutic since the fluid compress your lungs and cause you SOB (by taking the fluid out, the tension will be relived and youll feel better, but, the medical specialist will decide how long theyr gonna put the chest tube in) and if confirmed, the medical specialist will proceed to some other investigation like bronchoscopy and biopsy, etc.... and the bell rang Candidate 2 (Brisbane) Status: Passed Same as above! AMC Feedback: Shortness of Breath Station 7 40 or 50 year old lady came back to your GP, after seen by Nephrologist. She was recently diagnosed as having ESRF and neprhologist suggested that she needs to have Renal Dialysis. Blood test were done, results given, FBE: Hb- mild anaemia, but > 100 UEC: ESRF picture (urea 35, Cr >500, K 5.3, blah blah blah) CMP: ESRF picture Task: History, Tell the patient about Dx and DDx and Management. Candidate 1 (Adelaide) Status: Passed I was confused at first since the stem looks so so medical...... but the station is 7 I went in and greet the patient ( shes holding a tissue paper and theres glass of water in front of her-> making me even confused coz i think about breaking bad news about ESRF) and ask how shes doing. She said not good and she said she did not want the treatment. Q. Do you know your condition well Latifa? Yes Q. Do you understand that youll need the renal dialysis for the rest of your life? Do you want me to explain you more about the treatment? She said she understands well, but, she doesnt want the dialysis and she doesnt want to be machine dependent and dont want to go to the hospital 3 times/week. And also she said she was explained by the nephrologist about the treatment and again she repeats that she doesnt want the dialysis. Makes me confused again and I asked the examiner whether renal transplant is the available option. The examiner told me that do your best with the provided information. So, I talk to the patient that theres also option of kidney transplant and ill discuss with the nephrologist about that and let her know. The RP just repeat again that she doesnt want dialysis. I feel a bit frustrated and then, I decided to ask the psych question from that stage

since it is the psych station (if its not psych station, Im going to tell her that its her choice and well respect her choice and refusal of treatment, blah, blah, blah) Q. Well, Latifa, I understand that its frustrating to face this chronic illness and hear about the invasive treatment (actually, Im frustrated :P), but, before we discuss further, can I ask you some questions first and some of these questions are also sensitive and Id like to assure privacy. RP said yes. Q. Hows your mood? Not good Q. Feeling low or depressed? Yes Q. Do you have any idea to harm yourself or others or end up your life? No Q. Sleep? Difficult to fall asleep Q. Sex? Not interested in it Q. involvement of regular activity? Not anymore Q. LOA? Yes Q. any recent weight loss? Yes about 10 kg in 1 or 2 months Q. Insight, Judgement -> intact Q. No perceptual symptoms Q. SAD -> no Q. Home and Family -> supportive, but, RP doesnt want to be a burden Then, I explained the RP. Latifa, I know that you are facing a very difficult period of time and I understand that you dont want to be a burden to your family and you dont want the treatment although dialysis is the best possible option for you at this stage and with dialysis, your lifespan can be extended about 5 years and increase the quality of life to at life to your days. However, I think your condition is more serious than this. According to what youve told me, I suspect you are having a condition what we called major depression with somatic features, where you have the depressive illness and as the result of that your body is also facing difficulty in falling asleep and LOA, LOW, etc.... So, at this stage, I need to refer you to the psychiatrist to start you on antidepressant medication and also psychologist to help you cope with your stress. Also, I will arrange the religious and spiritual group to help you and also social worker involvement for your family since you are also worried about being a burden to your family. And, Ill also discuss with the nephrologist about the option of kidney transplant. Are you okay with the management? She said yes, and she repeated again that she doesnt want the dialysis. Then, I felt blessed that the bell rang at that time. I forgot to mention the possible differentials, but, I dont even want to think about this now. Candidate 2 (Brisbane) Status: Passed Same as above! AMC Feedback: Depressed Mood Station 8 - rest Station 9 Intern in ED, 55 yo man, epigastric discomfort, anxious, sweaty. Task: History for 3 mins, PE, Present the case to the examiner as if you are presenting the case to your registrar or consultant and most likely Dx and ask only one most important invs and interpret the invs to the examiner. Candidate 1 (Adelaide) Status: Passed

The RP looks anxious and SOB. So, I started by asking the examiner if my patient is vitally stable and he said yes and told me to start my task. Hi, X, My name is .... Im the intern working in this ED. From the notes I understand that you are having pain in your tummy, can you show me where exactly it is? Then, the RP point around the central chest. Then, I ask for pain Qs. Started about 2 hours ago at rest, Central chest pain, squeezy tight radiating to jaws, SOB+, Sweating +, N+, no V, No recent H&M, No fainting or dizzy spell, no weakness of limbs, not the first time having this pain. Had heart attack 2 or 3 years ago, did not have any invasive treatment or angiogram, doesnt know about Echo at that time. PMHx: DM, HTN, Hyperlipid - all present and on regular medications including aspirin and compliant No recent surgery or travel, no calf pain No family hx of sudden death, Hx of DM in mother I forgot to ask about SAD bcoz I needed to rush through my hx fearing that I would run out of time. Then, I asked the general appearance, vital signs (unable to perform postural BP). He is vitally stable at that stage. PR- regular. No pallor, J or cyanosis, JVP NE, Resp: normal, no crepts or added sound, CVS: normal, no mur mur, Abdo: normal. No calf tenderness or pedal oedema. Bedside UA: normal Then, I told my examiner that I would like to present the case. The gentleman Im seeing now is 55 year old Mr.X, who came in with 2 hour history of sudden onset of central chest pain, blah blah blah......... Hes got the significant background of previous heart attack 2 or 3 years ago, but, he did not have any angiogram done at that time. He also has co-morbidities of DM, HTN and Hyperlipidemia. Hes on medication for his diseases and claimed to be compliant. However, he did not have any history of recent travel or surgery. Examination-wise, apart from SOB, anxious and sweaty looking, hes vitally stable, JVP is not elevated and systemic examination is normal. Therefore, my provisional diagnosis at this stage is Acute Myocardial Infarction and Id like to arrange ECG. Then, the examiner asked me what are my points for diagnosis? Cardiac sounding chest pain, significant BK of heart attack, then, he interrupted me asking what are the risk to this patient. Age, Male gender, Previous history of heart attack, Comorbidities for CVS risk including DM, HTN and Hyperlipidemia (I forgot to ask about SAD and stress, but, look, hx taking was only allowed for 3 mins and I didnt feel guilty for that). The examiner said OK and showed me the ECG. So, I present the ECG to him again. This is the ECG of Mr. X, 55 yr old gentleman, sinus rhythm, PR around 90/min, No widening of QRS, ST segment elevation in II, III, aVF which are inferior leads and also in chest leads V1 and V2, anterior leads with reciprocal changes in I and aVL. Therefore, this is the case of ST elevation Myocardial infarction. Then, he asked me how old do you think the infarct is? Very recent within 6 hours. Why? ST segment elevation and hyperacute T changes. Then, he asked me what will the old infarct show? Q wave and he asked me to find Q wave in the ECG which Im not so sure. So, I just told him that I think theres 2 small square of Q in lead III and then, he asked me what does the Q wave mean? Im not so sure at that stage again and told him that previous infarction and he asked me what else and I really dont know more and lucky that the bell rang that time Candidate 2 (Brisbane) Status: Passed Same as above! But the examiner didnt ask detail about ECG finding. AMC Feedback: Acute myocardial infarction Station 10

4 wk old BIBM, crying, vomiting sometimes Task: History, PE, Dx and management. Candidate 1 (Adelaide) Status: Failed as expected This is the most wired station. When I entered into the room, the RP is middle aged lady who looked so so frustrated and tired (about to cry with red eyes) carrying a baby (doll). If this is psych station, I can tell from the look that this is postpartum depression/psychosis. However, this is paeds station. So, first, I congratulated her about the baby and asked whether shes coping well and has some support from home. She said her husband is supportive but, both of them are so tired because the baby is crying, vomiting all the time since birth. This is the second baby (first one is about 3 years old and well), planned pregnancy, no prolonged hospital stay, uneventful delivery. The baby was vomiting since birth varying 2 to 3 times or more, sometimes postfeed, sometimes, not, now more often. Vomitus: milk, not projectile, no bile, but, some blood curds/streaks present recently. Also, diarrhoea about 2 to 3 times/day. However, did the baby check with clinic which was normal growth. Babys still responding well. Had immunization at the hospital. Home situation; fine, no financial problem. The history is just wired. To my knowledge it doesnt fit into any of the possible Differentials -> infantile colic, irritable baby, GORD, HOPS or DA, ?child abuse PE: unremarkable, growth chart: normal, bedside UA: normal I dont know what to say at that stage. However, since the babys vomiting some blood, I thought I should refer to the hospital. So, I told the RP that it is reassuring that the babys growing well and PE was normal. However, since the babys vomiting some blood, Id like to refer the baby to the hospital. Then, the RP asked (looking angry), what will they do? The paediatrician will reassess the baby and if necessary some blood test and USS. But, Id like to reassure you that the baby might be having problem as simple as irritable baby. Then, she asked me why the babys vomiting a lot...... I felt so confused and I couldnt concentrate well at this station so, I couldnt give her the answer at that time, but, I told her thats why Id like to have the second opinion from the paediatrician and the bell rang..... felt relieved So So wired Candidate 2 (Brisbane) Status: Passed I started to explain the Mom that I suspect a condition called reflux but we still need to exclude other important causes. So, I would like to refer your baby to hospital. Then I reassured her dont worry too much because the baby is growing well and PE was normal. At Hospital, your baby will be reviewed by specialist and they will arrange required investigation like some blood test and USS and treat accordingly. She asked me why there is blood streak and so I explained that due to repeated vomiting, there might be tearing at the end of gullet. Then, she asked me why the babys vomiting a lot...... Bell rang! AMC Feedback: GORD with oesophagitis Station 11 25 or 30 yr old lady, came to your GP concerning about infertility. She had check up with gynae about 3 years ago for her period pain and invs had been done and turned out to be normal. Task: History, management

Candidate 1 (Adelaide) Status: Passed Hx for both patient and partner- all normal (no previous child or STI in both), apart from infrequent sexual activity: only 1-2 times a month (the RP cannot give the reason, why this happen). So, I asked her whether she knows about the ovulation period. She said she only heard about that, but, dont know exactly what it is and asked me to explain. So, I explain from the importance of timing for fertility and the need for the ovum to form the baby and I told her how to find out about her ovulation period (her period is totally normal and regular, so, day 14..... basal body temp, cervical mucous, blah, blah blah.....) For management, I told her that I will see her husband also and take hx, PE and possibly, semen analysis. However, at this stage, we can see how we go by timing the sexual activity d/r ovulation period. And Ill regularly follow you guys up and if no satisfactory progress, will refer to Gynae again for further management. Any other concerns? She said shes fine, but, one Q-> will I need to take some fertility pill? I told her not at this moment and the bell rang. Candidate 2 (Brisbane) Status: Failed Quite the same as above but I failed this station. I dont know why? AMC Feedback: Primary Infertility: unknown cause Station 12 55 + year old male patient comes to your GP for pain in lower limbs during walking. He has history of chronic smoking. Task: Physical Examination Candidate 1 (Adelaide) Status: Passed Burgers test is the main point they want Candidate 2 (Brisbane) Status: Passed Peripheral Vascular Disease (real patient) Reduced temperature, loss of dorsalis pedis and posterior tibial artery pulsation Burgers test (examiner asked me how to do Burgers test) Ankle brachial index (examiner asked me the normal value) Abdomen examination for any mass or lesion AMC Feedback: Leg cramps on exercise Station 13 - Rest Station 14 Police officer, ~30 yr of age, previously Dx as essential HTN, now came back for removal of suture, BP 165/100, stopped taking medication for 6 months (was on perindopril), recent blood test: UEC, Lipid, Sugar- Normal. Task: Assess patients compliance to medication and counsel the patient Candidate 1 (Adelaide) Status: Passed

She was diagnosed as having HTN while she was interstate about 3 or 5 years ago. At that time, she had difficult time period, lots of stress at work and family. She was compliant initially, but, because of the job nature, she started to miss the medication and experienced no difference and so, stop taking it. However, she did not suffer from the side effects of the medication. About 1 month after stopping the medication, she checked the BP at work place which was normal which reassured her and then, decided to stop taking it. She has family h/o of HTN in her mother, dx in later life. She thinks shes too young to have HTN and too young to be on life-long medication. Apart from stress, no other CVS risk factor, no symptoms of HTN at all. I dont even know how to manage it since she checked her BP after stopping meds, currently no symptoms, blood tests- normal. However, I started to explain her the risk HTN will impose on her and immediate risk of death or stroke and requirement to be compliant. I also refer her again to the physician for reassessment. Throughout that station, I didnt blame her or antagonize her wishes, but, also I didnt recommend her to stop the medication and also, I advise her on life style changes, stress relaxation techniques. Again wired station. Candidate 2 (Brisbane) Status: Passed Same as above! AMC Feedback: Non-compliance with anti-hypertensive Station 15 17 y o boy came to your GP, he is known to you and healthy boy. Now, seeking some advice regarding the following condition. Picture given showing acne. Task: History, Management Candidate 1 (Adelaide) Status: Failed I went in and greeted the patient asked how can I help him. He told me that he just came today because of the acne. He had that for about 2 years now and since he grew older, its quite embarrassing and so, wonders whether I could help him or not. I appreciated his concerns and asked him whether he tried some measures or medications regarding this. He said yes, the pharmacy gave him some cream and not working and thats been about a month now. Then, I asked him about HEADSSS since hes teenager and all are negative. Hes generally healthy and parents are supportive; however, he feels embarrassed about this and has difficulty in dating girls. NKDA. Then, I told him that I understands his concerns and understands thats very embarrassing problem for the teenager. But, I told him not to stress too much about it and we can sort this out. Then, I explained him that acne is mainly due to hormonal changes during pubertal life or sometimes unknown reasons. It is not related to hygiene. Please dont stress yourself since it is not your fault or due to your hygiene. Just relax and staying cool is the first measure and avoid the facial scrubs coz that might erode your skin. And do you know the name of the cream the pharmacy gave you. He said yes, I wrote that down for you, which is Benzoyl peroxide (im really not familiar with that). However, I advised him to stop it since its been a month and obviously not doing its job and it might also cause some erosions (im totally not sure with this thing, please check yourself). Then, I told him, sometimes, acne is associated with some infection, so, we can also try antibiotics. He looks interested and asked me what medication. Again, Im not so sure, so, I told him that Ill check with my senior, but, I think its doxycycline. Are you okay with that? He said totally fine. And Ill also follow you up in 1

weeks time to check the progress. If not improving, Ill refer you to the dermatologist for some advices and he would start you on steroid cream (again, not sure). But, for the last resorts, we can also give you retinoic acid and youll be fine with that because it has some adverse effects on girls, but, you are a boy and should be fine. Then, I asked him any other concerns, and he said will he have scars afterwards. Good question! Totally not. You might have very tiny scars, but, you wont be disfigured at all. Then, he asked me his mum said the acne is due to chocolate and he wonders about that. So, I told him its just depend on personal preference, if you like it, you can eat it and theres no documented effect of chocolate on acne. Then, the bell rang and again, felt relieved. Candidate 2 (Brisbane) Status: Passed I reassured him that his condition is called acne vulgaris and most acne usually settles by age of 20 years. I advised him to stop Benzoyl peroxide. He asked me the possibility of scars on his face after resolving his problem and I answered highly unlikely. Then, I told him I want to give him some antibiotics. At that time, the examiner asked me what antibiotics will you use and I said Doxycycline (actually Im not sure ) Then I explain about general measures (Diet to avoid too much chocolates and spicy foods as there is some casual relationship in some people and how to wash face gently with normal soap). He looks happy. And Ill also follow you up in one week time and if it is not well improved, Ill refer you to the dermatologist specialist to get advices. Bell rang! AMC Feedback: Acne Vulgaris Station 16 HMO in rural hosp. 6 wk old BIBD, problem with feeding for 1 week, poor suckling today. Task: History, PE, Mgt Candidate 1 (Adelaide) Status: Passed First born child, delivered at home, no immunization at delivery, previously well since after birth. Poor feeding for a week, now poor suckling today. No fever, snorty nose, not sure about cough, seems to struggle for breath today. Reduced amount of wet nappies (only 1 for today, which is not really wet). Looks uninterested and lathergic. No family problem, No possibility of abuse. T 36.5, RR 70/min, O2 sats: not done, tracheal tug and IC indrawing +, cannot hear the heart sound well bcoz of fast breathing, bilateral basal crackles + Hepatomegaly 5 cm So, I headed towards Respiratory Distress, most probably bronchiolitis, however, theres also possibility of pneumonia. Put on O2, put a line in, take blood including cultures, talk with paeds specialist in tertiary hospital, arrange CXR and arrange for transfer. In the mean time, O2 and IV Abx if the specialist suggests so. Also, D/W father about importance of immunization. I also told the father that he or his wife or both of them can accompany the baby to the tertiary hospital. The examiner asked me, what if you observe the baby in your department for about 4 hours and no signs of improvement, what would you like to consider on examination? I really didnt know what she meant at that time. Then, I started to procrastinate around the possible diagnosis. Then, the bell rang. Later on, I realized that she was asking for Heart Failure as DDx. However, I couldnt really think that much at that time and also, it was my last station (just an excuse :P) ?? Heart Failure ?? Bronchiolitis or LRTI

Candidate 2 (Brisbane) Status: Failed PE: T 36.5, RR 70/min, O2 saturation: not done, Chest indrawing +, Both systolic and diastolic murmur +, bilateral basal crackles + Hepatomegaly 2 cm So, I think it is heart problem (heart failure) and so explaining about this. Admission and manage as heart failure. I failed this station so please check other recall. I failed as I did not tell differential diagnosis like bronchiolitis! AMC Feedback: Respiratory Distress Station 17 60 yo man planned for caravan trip in 4 weeks time. Came to you for check up and you found abdominal mass and arranged USS which confirmed AAA 5.3 cm. The patient came back today for result. BK h/o HTN on perindopril. Task: Explain the result and management plan to the patient. Candidate 1 (Adelaide) Status: Passed This is exactly the same as previous recalls. This is my first station and my examiner is Dr. Wenzel My voice was shaky for first minute, but, I managed to control myself Candidate 2 (Brisbane) Status: Passed When I enter the room, there is 60 year-old gentleman sitting there. But I explained his condition. Your biggest blood vessels which distribute the blood from your heart to all parts of your body now become dilated. We dont know the exact cause but there are risk factors such as family history, age, smoking, hypertension and diabetes. There is the risk of rupture especially if it is more than 5 cm. He asked me can he go to trip and do operation after that. I told him that I wont recommend you to do so because there is the risk of rupture. If you go for the trip and rupture, your wife call ambulance and even if we do urgent operation, the chance of successful operation is very low (just 10%). I told him that if we operate now, the chance of success is 95%. Then he wants to know whether he will be ready to go for the trip in next 4 week if he did the operation right now. I told him that, I am not sure. It depends on your condition. But he wants yes or no answer. So I told him that, I am sorry I dont know but I can ask the surgeon and can tell you back. Then he asked me what operations they will do? I told him that, it might be putting a stent from your vessel at your thigh or they might do open operation and put a graft at the site of dilated partBell rang! AMC Feedback: Abdominal Aortic Aneurysm Station 18 - Rest Station 19 30 yo nurse came to you where you work as a GP. She was seen by your colleague a week ago. She has repetitive intrusive and irresistible thoughts of washing her hands very frequently because she thinks her hands might be contaminated and transmits infection to patients or family member. PE and invs were done and she was diagnosed as having OCD (they give the

definition of OCD in the stem). She is feeling anxious and frustrated and now thinks whether she can have some time off from work. Task: History, Management plan Candidate 1 (Adelaide) Status: Passed I went into the room, greet the patient. She was quite a nice lady, wearing rubber gloves. C: Hi, howr u going, Geraldine? RP: Im fine, but, I feel very anxious and frustrated about this problem and as a nurse, Im afraid I might transmit infection to the patients. I couldnt help thinking about that and keep on washing my hands several time and keep on wearing the gloves. C: What area of speciality r you working? RP: just general (actually, she wasnt prepared for this Q) C: So, do you think its rational to keep on washing the hands or this idea just drive you mad? RP: I know its irrational, but, I just cant help. C: Okay, I understand. Its kinda very frustrating, isnt it? RP: yes. C: However, its really good that you know its irrational and try to resist the thought and sought some help. Apart from this thought, do you have other preoccupying thoughts? RP: no C: do you used to be a worrier/anxious person? RP: sometimes, but, lately, worries a lot. All the other psych Q, she responded No. Insight (+), Judgement (+), No suicidal idea, lives with family and husbands very supportive and concerns about her problems, no financial problems Then, I started to explain about the management plan. Well, Geraldine, the problem youre having, which is OCD, THEN, the RP interrupt me asking What is that problem? I was surprised that the RP did not know the problem yet coz I thought she already know her problem and just come back for follow up and asking for some time off. However, I explained her what is OCD again. She was quite okay with the explanation. However, shes afraid that she might be crazy. So, I reassured her. This is just the kind of anxiety disorder and youve got your insight and judgement and Id like to reassure you that you are not going crazy at all. (Then, she looks relieved.) For the management, Id like you to relax and reduce the stress, dont bottle things up, talk with your friends or husband who are supportive and caring. Also, Id like to refer you to psychologist and do some CBT which is a talking therapy and hell also do special procedure called exposure and response therapy. Then, the RP looked interested again and asked me what is that. I told her that he will create a simulative environment and make you exposed to the situation where you would like to wash your hands very much and hell help you to get over that situation. Also, I will regularly follow you up and if your condition is not responding well, I can also refer you to the psychiatrist to start you on antidepressant, SSRI (i asked her whether she knows the medicine or not and she said she heard about it) and with the combination of SSRI and CBT, the prognosis is pretty good. Finally, I asked her whether shes happy with the plan and she said yes, and then, she asked me whether that will be beneficial if she has some time-off from work, which Ive totally forgotten. I told her that its a good idea to have a timeoff and have the therapy, refresh herself and that will be beneficial to herself and also, the family, colleagues and also the patients coz if shes so stressful and anxious at work and home, that will make the situation worse and then, she looked happy and the bell rang. Candidate 2 (Brisbane) Status: Passed Same as above!

AMC Feedback: OCD Station 20 25 or 30 year old lady came to your GP for AN care advice. She is wondering whether she could have your care in part of joint care with the hospital outpatient clinic. PE: normal, BMI 40. Task: History, Counsel the patient regarding AN care advice Candidate 1 (Adelaide) Status: Passed The stem sounded simple; however, I went into trouble with this station. I greeted the patient and asked her how can I help her. She said she would like to have some AN care advice from me. So, I asked her whether she already checked up with hospital OPD and she said no, she said shes planning to have a joint care. I asked her whether shes pregnant now and she said no. There are also no signs of pregnancy on history. So, I started to ask 5P Period: about 13 yrs of age, been regular, normal amount, no dysmenorrhoea Partner: married, no h/o STI, stable relation Pills: no contraception and then, I asked her how long theyve been trying to get pregnant and then, she said about 12 months now and couldnt get pregnant. (At this stage, I was shocked wondering whether is this station ANC or Infertility.) No previous pregnancy or miscarriage, Normal pap smear Sexual activity about 3 times/week, knows briefly about ovulation period. Partner has previous relations, but, not known to have STIs or children or orchitis or injury. Then, I dont know what to ask for since if it is infertility station, I couldnt find the cause apart from obesity, however, she doesnt have other features of PCOS (regular period, no acne or hirsute), and her general health is perfectly well. So, I started to talk to her that to have the AN care, if you need to be pregnant and youve been trying for 12 months and no success. So, I would like to refer you to the gynaecologist to check you up and he/she will reassess you and will do some USS of the womb and some blood tests. And also, I would like to see your partner and assess him and do semen analysis. Also, I advised her to reduce the weight because she falls into obesity group and the RP didnt seem to be happy with weight reduction. However, the bell rang at that time and I felt relief because Im not so sure about the task and theres no hint from the RP or examiner about my task. I didnt even have the chance to talk to the RP about the AN care process when shes pregnant or didnt have the chance to arrange the urine pregnancy test. With this station, the RP is not hinting at all and she just answered in very short sentence regardless of whatever I asked her and I needed to dig very details to have the information I want. When I walked out of this station, I was pretty sure that I would fail this station. ANC advice + infertility (?obesity and infertility) Candidate 2 (Brisbane) Status: Passed Same as above! I explained her also the effects of obesity especially on pregnancy and fertility. So, I advised her to reduce weight. I explained Life style modification, Medications and also surgical methods to reduce weight. AMC Feedback: Pre-pregnancy Counselling of an obese woman

After I got the feedback, I cant believe that I pass all O&G :P..... However, I feel blessed that I passed one Paed station. Anyway, at the end of the day, the main concern is to pass the exam, which doesnt necessarily reflect how good you are in actual working environment (just reflect how well you perform in that 8 minutes of agony) and get yourself fully registered. Id like to wish everyone good luck with the AMC 2 exam. I also would like to remind you all that this recall is just written to the best of my knowledge and I cannot guarantee that the information are correct. But, I do really hope that this will be of some benefit to the candidates preparing for the AMC2 exam. With this opportunity, I want to thank my family and friends, especially my study partners (Dr. Aung and Dr. Myo) and the last, but not the least, Dr. Wenzel. CHEERS!

Retest 28 May 2011 Melbourne

Station 4 60 year old man came to consult you, his GP regarding to screen for prostate cancer because his 75 year old friend at local rotary club had been recently diagnosed prostate cancer. Task: History, counsel and discuss the risks and benefits of prostate cancer screening

This is my first station; I was a bit uncertain when reading the stem, which was a bit unfamiliar to me (no in my recollection of recalls). The history showed that his father also had prostate cancer but died from a heart attack at the age of 80. The patient has some symptoms of BOO but no other risk factors except his fathers prostate cancer history. Then I wanted to get the correct dx by asking for PE findings but the examiner told me to stick to the task so I told the patient that I needed to examine him to assess the size, shape, consistence and presence/absence of nodules. Then I explained that prostate cancer is a slow growing tumour and people dies with prostate cancer rather than died from it. Currently RACGP does not recommend screen for prostate cancer but considering you have a family hx of prostate cancer so if you are worry we can perform simple blood test (PSA) and US. He asked me what the risk for the screening, I explained to him that DRE does not hurt although a bit discomfort for him, PSA is a simple blood test and US just a probe to give away some signals to detect the prostate gland. I had not mentioned about biopsy because I did not want to alarm the patient unnecessary. There were not questions from the examiner and the patient was happy.

Station 5

25 year old woman had a CS over two years ago now is 7 weeks pregnant, she came to enquire whether she can have a normal vaginal delivery. Task: Hx, PE and answer her questions

After went through the first station feeling reasonable ok, I was a bit comfortable when I was reading the stem. I started the history by congratulating her second pregnancy, then asked about the previous pregnancy especially the details with the CS, which was done with low vertical cut over 2 years ago in a tertiary hospital with no complication because of failure to satisfactory progress of labour. No other surgery history and contraindications for vaginal delivery. PE showed nothing except an old horizontal scar in her supra-pubic area. She then asked whether she can have vaginal delivery although admitting it was too early to discuss it. I then explained that there is no contraindication for vaginal delivery and point out the risks associating VBAC, therefore depending on the progress of pregnancy, vaginal delivery can be attempted but this must be done at a tertiary hospital where emergency caesarian section facility is available and no oxytocin should be used. The patient was happy.

Station 6 Middle age woman with long term diabetes recently developed numbness on her lower limps Task: Perform relevant lower limp examination It was an old recall but usually including further investigations and management. When I came in the lady was already lying in the bed with the leg exposure up to knee level, I asked for permission and said I already washed my hands as there was no antiseptic gel or tap & sink around. When I asked the patient whether she felt any pain or numbness, she did not understand or unexpected my questions but the examiner told me to carry on the examination. No abnormality on inspection but no pulses in both dorsalis pedis and posterior tibial but popliteal pulse presented. Proprioception only presented in knee level and other sensory tests (pain, light touch, vibration) lost just above ankle level. There were two tuning forks with the 256 Hz obvious on top while the much smaller 128 Hz one was covered by cotton wool and hammer so I needed to look for it. I finished the examination early but the examiner asked what are you going to do now? I asked for clarification: Do you mean in term of investigations and management? No, just in term of physical examination I was sure then as I thought I had done everything in the lower limp so I answered I would like to check her eyes for diabetic retinopathy. The bell rang.

Station 7 50 something old man has been suffered abdominal pain, constipation, diarrhea and lost 5 kg in two months, investigations including gastroscopy and colonoscopy are normal. Task: hx dx and management

I forgot this is psych station when I was reading outside the room so I was thinking what dx would be responsible to these symptoms if both gastroscopy and colonoscopy are normal, pancreatitis? liver cancer? Etc but could not come up any answer then I suddenly realized the magic 7 just before the bell rang for entering the room, I was still worrying about his weight loss when I got in the room. I took the history by asking HEADS, depression and psychotic questions as well as substance abuse and general health. Because of the stress from work as a teacher a couple months ago, he, now retired, presented predominately anxiety symptoms with sleep disturbance, weight loss and reduced appetite. His mood is ok, kind of isolation rather than low and libido & interest slightly reduced but no suicidal or homicidal or substance abuse or psychotic features. I was not sure about the psychiatric dx at that time as it did not fit into particular category such as major depression, generalized anxiety disorder or adjustment disorder according to DSM IV criteria. So I decided going to the safer option and told him that he got an irritable bowel syndrome. I then explained the body and mind relations, reassure the benign nature (not malignancy) of the condition and offer counselling, CBT, reading materials and regular following ups.

The role player was reluctant to give clear answers to my questions such as life is not worthy living, hurting yourself or somebody else, how is your mood like, etc? When asked whether he was happy with the plan of management, his reply was Only when you say so! so I did not feel well when the bell rang. I talked to a few of my fellow candidates after exam with the same or different role players and they all said the anxiety was the main feature so I was surprised that the feedback of Major depressive episode.

Station 8 Rest station Station 9 Book case 132 (page 671 and 700)

I just did this case in the trial exam one week before the exam so I was confident with the task. However after asking me whether I am understand my task, the female examiner did not want to give me the sheet with the results of the pulmonary function test by asking Do you really need this sheet? I then started by explain the test results using that sheet and some drawings but as soon as I completed the spirometry test the patient immediately asked What have I got, doctor and what can be done? I quickly mentioned what the lung volumes and diffusing capacity mean then went on to explain the dx, its causes and management according to the book and John Murtagh. Then the patient asked me whether she has a lung cancer, I reassured her that as far as she continues staying off smoking her chance of developing cancer is reduced but I forgot to mention to do CXR to rule out that unlikely possibility if one has not been done in previous 12 months.

I then asked the examiner whether I should show the patient how to use the medication before the 6th minutes (she did set the alarm for the 6th minutes) and she said yes. There was a large cone shape spacer packed in a box, a bag of mouthpiece resembling the empty toilet rollers and Vetolin puffer on the desk. I demonstrated the 4x4x4 method with the spacer. I completed the task before the bell rang but I was worrying this station about the exam as I forgot to mention to do a CXR, which could be the critical error.

Station 10 Book case 47 (page 222 and 248) but includes taking hx, PE and Mx

The patient was lying on the bed but the examiner was at another side of the room so I bended over at the bedside to take the history that was similar to the one in the book. I was not sure whether I need to perform PE so I only took a very brief hx. The examiner waved me over to another side the room then asked me what I want to know in PE, so once I said I would like to exam the lumber area and low limps but starting with inspection. He then gave me all the examination finding from inspection, palpation, movement, special tests and tone, power, reflexes, sensation and coordination. The patient has slightly scoliosis to the left and limited movement in all direction of lumber joint with local tenderness in L5 and S1. SLRT only 30 in the left and left ankle jerk diminished and sensory loss in lateral side of left feet.

I then returned to the bedside to tell the patient that he got a S1 disc prolapsed and explained it by drawing a diagram. He then asked whether further test is required and No this moment. I will give you some analgesic for pain relief, you need to rest in bed for a couple days but need to get up to move around and start physiotherapy after that I then reassured him that most of these cases will resolve completely without serious consequences. He then asked how long he will be away from work, I said I will give you a medical certificate for a couple days but I will see you again then to reassess your condition at this moment you are definitely off work for a couple days. There was no questions from the examiner and the patient seems happy.

Station 1 Middle age woman complained visual disturbance for three times since last weeks. She has hypertension and hyperlipidaemia and on a list of medication including aspirin, statin and ACE inhibitor or -blocker. She also had an angioplasty/bypass before and family hx of DM Task: Hx PE & Management

In history, she has amaurosis fugax (curtain coming down) but no other symptoms such as verbal problem and weakness of limps. PE showed BP 150/100 P100/min but regular, second degree hypertensive changes in fundoscopy and carotid bruit. So I explained the dx is TIA and requires urgent admission for further investigations such Dopple carotid US, CT/MRI of the brain, ECG, TOE and blood tests. The role player asked Will it happen again? Many patients who have TIA will have stokes soon after TIA, this is why you need to go to hospital immediately and to prevent it happens. Do you have any other concerns? No, doctor, thank you very much!

Station 2 30 year old woman who has an 18 months old child with Downs syndrome now is 7 weeks pregnant. She and her partner went to geneticist and were told that the chance of having another Down syndrome child is 1 in 100. Task: Answer the questions from the mother

I was reasonable comfortable when I was reading the stem because we had argued a few times for this case since it reappeared in March 2011 exam. What does 1 in 100 mean? I explained the chance of developing Down syndrome in

normal pregnancies in different ages and pointed out she is about 10 times higher risk than the normal pregnancies. She was a bit worry Then I explained the screening and diagnostic tests and their benefits & complications. She seems not very happy so I wanted to find out what exactly her concern is, i.e. whether they do not want another baby with Down syndrome that was happen in old recall. She told me that her partner does not want to talk about this matter but she still want another baby who with Down syndrome or not. I offered to see her partner, or joined consultation and simple literature to take home for her and also mentioned the support group for Down syndrome as well as ongoing support/following up. She was happy and no further questions.

Station 3 Rest, I finished my exam with a glass cold water and good feeling about the exam.

It was such relief when the result came out. Everyone I know who took this exam passed. All the examiners except one who did not shave my hand and the role players except one who is major depressive are very helpful and encouragement comparing to my full exam in another place. So just like Dr. Wenzel said that retest is not a bad result although you have to go through PTSD first.

I would like to express my gratitude to Dr. Wenzel, all my teachers in different hospitals, all my lecturers in VMPF long course, my various study partners, and of course, family and friends for their generous supports and ongoing assistance. I wont be here without them!

Scenarios AMC Perth 21.05.11, written in retrospect Biliary colic (passed) RUQ pain. History and clinical findings given. Task was to explain the most likely diagnosis (biliary colic). Prior to starting I offered the patient some pain relief. I explained all the possible differentials and then management. The patient wanted to know the plan from here (could he be discharged or not, how long would he have to be off work.) Chronic liver disease (passed) Only real patient in exam. Very clearly hat liver disease. Task was to examine the patient.

Clinical and U/S confirmed IUGR (passed) Explain causes and management. I could not remember all causes and if it was necessary to further investigate. This case is a bit of a blur. I managed to be reassuring to the patient (at least I think I was) and list quite a few causes. I made sure she was properly followed up. Croup (passed) Fairly straight forward. Explain management. Question from the examiner: Could it be something else? Delirium (passed) Aboriginal woman refusing dialysis. Task was to do a mini mental test and then manage. Outcome of the mini mental test was that she was delirious (clearly disoriented). Patient appeared depressed. I completely forgot to ask about drug history. I managed to convince the patient to be admitted and mentioned in the last minute that she needs further investigations. I think I could have handled this case better, as I ended up telling the patient she needs a psychiatrist (which is only appropriate once an organic cause for delirium is excluded.) I did think this all the time but did not voice this to the patient. It must have looked quite bad ,-) Diabetes mellitus in four year old (passed) Setting in GP practice, young kid presents with bed wetting (if I remember rightfully). Through history it becomes clear there is polydipsia. Asking for investigations: Urine dipstic positive for sugar. Patient clearly needed urgent referral to hospital. Father asked a few questions, including what would be done in hospital and would his son need insulin forever. Dyspareunia (passed, unsure why) I clearly struggled with this one and thought I had failed. Setting was in a GP practice, task to take history and ask for examination findings. Young recently married woman, previously no problems with intercourse, now dyspareunia. Implanon some time ago. I think it was vaginitis secondary to chronic candida infection. I failed to illicit this in the history and could not put 2 an 2 together after I was given the examination findings (white specs on vaginal exam) End of life request (passed) As per AMC book, exactly the same.

Headache acute (passed) Setting ED. Patient had sudden onset of headache. History photophobia. Exam: BP normal. Febrile, some neck stiffness. No rash. Otherwise normal neuro exam. My diagnosis was meningitis. (I assume I was right given I passed). After making sure the patient had adequate analgesia I explained further management (my approach was CT and then LP. Antibiotics as soon as possible after tests, unless there were significant delays to get tests organized). Obesity (passed) GP practice. Routine follow up for asthma. You the Gp notice the child is more than chubby. Task was to chart growth charts and explain significance to father. Child had gained a lot of weight and was above the 95% percentile (prior was normal). From the history it was evident that the Kid loved Take Away Food and had no exercise (preferred computer games), so the weight gain was probably life style related. I suggested a referral to a dietician and lifestyle changes. The dad asked me but what about his glands. Not sure if I was required to do further testing.... Pancreatic pseudocyst (passed) GP, patient with bloating. Recent procedure mentioned in stem. Taking the history it became evident, that patient had recently undergone ERCP. After finishing the history and examination, the examiner asked me to interpret a coronal CT image which showed a LUQ collection. I called it that. I feel a little ashamed to admit, that I did not click: ERCP + collection in LUQ = Pancreatic pseudocyst. I thought it could also be an enlarged spleen. I think it would have been fairer to have multiple slices of a CT to be able to localize neighbouring structures (I would have been able to spot the spleen). I passed, unsure why. Psychosomatic chest pain (passed) Lots of investigations done, no cause found. Turns out patient has sister with heart trouble. Took a psychosocial history and asked for depression. Explained it is highly unlikely this is somatic chest pain, but would refer her to a cardiologist if she wanted this. I further made clear, that I think a lot of what was happening in patients life was to do with lots of social stressors, including sisters illness. Sore throat and rash (passed) Take history and explain reason for rash. Image shown showed maculopaular rash, onset after taking Amoxicillin prescribed for sore throat by GP. Diagnosis. Drug reaction to Amoxicillin on a background of likely glandular fever.

Transient visual loss due to thrombembolism (passed) Task take history and examine. Everyone do not forget to examine the cardiovascular system after doing your neuro exam. Turner syndrome (passed) Primary amenorrhoea in a 18 year old. Appearance scribbled down on a piece of paper (as clearly the role player did not look this way): Short stature and something else. No webbed neck. Task to take history and ask for investigations. Thus far not too hard. But the big question is, how do you explain Turners to the patient. How do you manage (do you give hormones, can a person with Turners become pregnant?). I was not sure, especially about pregnancy, I thought maybe there was a new fancy way of hormone treatment to facilitate pregnancy, even though I seemed to remember pregnancy was not possible. Lets put it this way, I do not think I came across cleverly.

Das könnte Ihnen auch gefallen